I&D

Pataasin ang iyong marka sa homework at exams ngayon gamit ang Quizwiz!

meningococcus a.k.a. Neisseria meningitidis vs. Streptococcus pneumoniae?

Gram negative diplococci Streptococcus pneumoniae = Gram positive diplococci Both can cause bacterial meningitis. Haemophilus influenzae, also called Hib can too! (Gram-negative rods)

Which pathogens have LPS?

Gram-negative bacteria (that whole pic is the LPS - Lipid A is just part of it) Innate immune system PRRs recognise LPS

Why does the parasite Trichomononas vaginalis, which causes an STI, not form a cyst-like structure like most other parasites?

It is never exposed to the outside world - just goes from 1 mucosal surface to another - so no need to invest in building a cyst around itself. But it is a flagullate protozome!

What is the mechanism of tissue damage with TB?

Lymphoid cells release toxic substances which damage tissue! It's not the bacterial toxins damaging the tissue themselves...Instead, the bacteria cause macrophages to form granulomas. These granulomas can liquefy and cause bacteria to spill into the lungs to cause large cavities to form active TB. The bacteria breed in the cavities. Thus, the tissue destruction results from cell-mediated hypersensitivity.

What does lysozyme do?

Lysozyme splits peptidoglycan in bacterial cell walls. The peptidoglycan layer contains alternating molecules called N-acetylglucosamine and N-acetylmuramic acid. These molecules form a strong glycan chain that act as the backbone for the cell wall. The link between the N-acetylglucosamine and N-acetylmuramic acid is cleaved by lysozyme

What are M cells in germinal centres of lymphoid tissue?

M-cells ('microfold' cells); these are specialized antigen-transporting cells. Foreign material, including bacteria, is taken up by the M-cells and passed on to the underlying antigen- presenting cells and T- and B-lymphocytes of the Peyer's patches.

Tissue damage by GAS a) What products are responsible for tissue damage? b) What is the mechanism of tissue damage?

a) Streptolysins S and O are haemolysins that are responsible for the beta haemolysis of GAS when it is grown on blood agar. However, they also lyse leukocytes, platelets, and likely other host cells. b) What is the mechanism of tissue damage? Bacterial enzymes destroy host cells --> Components released from damaged cells and bacterial cell wall components (PAMPs) cause an acute inflammatory response.

Name 3 pathogens that we still don't have an effective vaccine against

There are still no effective vaccines for HIV, Tuberculosis and most major parasitic diseases

B-lactams have NO effectivity against which bacteria?

atypical organisms w no (or v thin) cell wall • Mycoplasma • Legionella • Chlamydia

What does neuraminidase (surface protein on influenza virus) do?

When a new virus is produced, the neuraminidase shaves off/cleaves the sialic acid so the virus no longer sticks to the host cell & go to infect other cells

Hair increases ...... and ..... on skin surface

humidity and temperature!

Pathological fracture

in patients who have tumours in their bones - tumour eats away at bone — could be from a cancer in x-ray will see low bone density - this area of skin will be mushy

Very common autoimmune disease in old people

the classical example is giant cell arteritis which is a T cell-dependent granulomatous vasculitis of mid- and large-size vessels. The vasculitis does not manifest before the age of 50 years; its annual incidence continues to increase up to the eighth decade in life.

Mutations in which gene are most commonly associated with severe combined immunodeficiency (SCID) A. RAG-2 B. ADA C. RAG-1 D. JAK-3 E. ɣC (gamma C)

ɣC (gamma C)

What are the physiological characteristics of mycobacteria?

• Bacilli (rod-shaped) - BUT if they shed their cell wall they r in a spheroblast form • Thick waxy coat surrounds cell wall -> means highly resistant to germicides and drying

What is the structure of Mycobacterium tuberculosis?

• Cell wall contains mycolic acid & arabinogalactan ◦ Mycolic acid makes up an outer 'mycomembrane' • Acid fast bacterium because the cell wall mainly consists of hydrophobic mycolic acid

Viral tropism

the specificity of a virus for a particular host tissue, determined in part by the interaction of viral surface structures with host cell-surface receptors.

Antibiotics (define)

Substance *produced by a microorganism* that in small amounts inhibits the growth of another microbe.

What is the difference between the FAB and the Fc regions of an antibody molecule?

The fragment antigen-binding, Fab, which comprises the arm of the antibody and includes the variable region which differs from one antibody to another. The variable region gives the antibody its specificity and recognises a particular antigenic 'shape'. The precise part of the antigen that is recognised by the antibody is called the epitope. The Fab region of the antibody contains the variable domain and the first constant domain (CH1) of one of the heavy chains and all (VL and CL) of one of the light chains. Each antibody unit has 2 arms and therefore 2 Fabs. The 'fragment crystallisable', Fc, contains the biological activity effector part of the antibody. It is composed of the remaining sections of the 2 heavy chains (CH2 and CH3 for IgG, IgA and IgD; CH2, CH3 and CH4 for IgM and IgE) and is where the molecule 'fixes' (binds and activates) complement and attaches to Fc receptors on cells. The constant region of the heavy chain determines the antibody class and is structurally adapted for a particular biological activity in each different class. Additionally, the different classes of antibody function best at different sites in the body.

Bacteria divide by binary fission. What do we term the time between one division and the next? Most bacteria grow on simple blood agar, although some are fastidious & require special growth factors. What is special about chlamydia?

"Generation time" - ranges from 20 mins to 24 hours. Chlamydia cannot grow on artificial media. It must be cultured in eukaryotic cells e.g. fibroblasts. They have a growth curve similar to VIRUSES

Adhesins are grouped together on the surface of bacterial cells in structures known as (?)

"fimbriae"

X-linked agammaglobulinaemia (primary immunodeficiency)

(defect in the Bruton's tyrosine kinase, Btk, gene) where no mature antibody-secreting B cells are produced.

Difference between central & peripheral tolerance?

(i) Central tolerance by clonal deletion: (ii)Peripheral tolerance by deletion and inactivation (anergy): Anergy = absence of costimulatory molecules Peripheral tolerance prevents harm from any self-reactive cells that manage to escape deletion in the primary lymphoid organs. Such escape can occur because, for example, not all self-antigens are present in the primary lymphoid organs. Lymphocytes in the periphery are normally kept in an unresponsive state through clonal anergy which is the result of the cells not receiving the pathogen-triggered costimulatory signals necessary for activation of the cell. Potentially autoreactive lymphocytes can also be suppressed by regulatory T-cells.

Wiskott-Aldrich syndrome

(mutation in the WASP gene on the X chromosome encoding 'Wiskott- Aldrich syndrome protein') in which there is a defect in cytoskeletal organisation, particularly in lymphocytes and NK cells. - when cells get signalled to migrate to area of infection they can't move through the cytoskeleton because it isn't flexible enough

X-linked hyper-IgM syndrome

(mutation in the X-linked genes for either CD40 ligand or the signalling molecule NEMO, or various autosomal genes including that encoding CD40) in which, due to a lack of T-cell help via CD40-CD40L interactions, B-cells are unable to undergo CLASS SWITCHING from IgM to IgG and IgA and therefore have no SECONDARY humoral immune response.

Define epidemic & pandemic

Epidemics are peaks in disease incidence that exceed the expected rate for that particular infectious disease. A pandemic is a world wide epidemic, e.g. the recent swine flu pandemic.

3 main functions of lymphoid tissue

1.Separate T and B cell areas for specialised differentiation pathways 2. slows cells down to facilitate cellular communication 3. Lymph node acts as mesh/sieve (made of collagen fibers) to trap antigens (for long enough for them to be recognised by the immune system)

What are some ways parasites cause disease?

1. Disruption of normal physiological function e.g. Giardia: uptake and metabolism of bile induces malabsorption 2. Invasion of host tissues e.g. trypanosome- induced cardiac failure OR Toxoplamsa & Leishmania invasion of macrophages disrupts immune response 3. Physical presence e.g. Ascaris blockage of gut

3 diff antibiotic resistance genes bacteria may have (usually found on the plasmid)

1. Betalactamases (enzymes which degrade beta-lactam antibiotics) e.g. in E Coli 2. Produce EFFLUX PUMPS e.g. bacteria uses the pump to pump out tetracycline outside of the cell! 3. Genes which modify the antibiotic binding target e.g. MRSA modifies penicillin-binding proteins so the pen can't bind to the protein in the bacteria anymore!

A. Which early genes are transcribed BEFORE virus replication? B. And which LATE genes are transcribed post replication?

1. EARLY - before replication : reverse transcriptase, DNA-dependent DNA polymerase 2. LATE - post-replication: structural genes e.g. capsid

During B cell antibody class switching in the germinal centre, what classes do the antibodies switch to?

From IgM to IgG (or IgA or IgE)

What will the ultimate treatment for primary immunodeficiency be?

Gene therapy - where the identified faulty gene is replaced in the patient's stem cells with a 'normal' gene; this has already been used in some forms of SCID (gammaC, ADA).

Name 2 important clinical applications of detection of viral nucleic acid

1. Detection and quantification of HIV, hepatitis B and C virus nucleic acid in blood, i.e. viral load, used for monitoring antiviral therapy. 2. Nucleic acid sequencing to determine viral susceptibility to antiviral drugs (genotypic resistance testing).

Genome of the Hep B Virus?

- Partially double-stranded DNA - circular genome

Probs with the influenza vaccine for old people?

- efficacy of only 30-40% in protecting elderly patients from disease But imp. cos : the systematic inflammatory response induced by influenza propagates many age-related inflammatory pathologies, particularly CVDs. Vaccinating against influenza is associated with decreased incidence of myocardical infarction and stroke. Influenza infection may accelerate atherogenesis, resulting in death months after infection.

Innate hypersensitivity

- toxic shock from Staph aureus & Strep pyogenes - acute respiratory distress syndrome form LPS in gram negative bacteria Types 1-5 = adaptive immune response. However, overzealous innate responses can also lead to pathology. Some infections, e.g. with Staphylococcus aureus or Streptococcus pyogenes, can sometimes provoke a toxic shock syndrome involving excessive release of TNF, IL-1 and IL-6. Another example of innate hypersensitivity is the acute respiratory distress syndrome which is associated with Gram-negative bacteria and primarily due to the lipopolysaccharide (LPS) endotoxin provoking a massive invasion of the lung by neutrophils. Chronic activation of innate immunity, especially macrophages is now believed to drive the pathology of many of the common chronic diseases associated with ageing, including Alzheimer's, Type II diabetes and atherosclerosis. Indeed a generalized increase in inflammation may be a key component of the ageing process.

If it ends in -mycin it inhibits...? Name some mycin sub groups? If it ends in oxacin? If it has "sulf" or "meth" in it?

-mycin = inhibits protein synthesis Macrolides (50s), Aminoglycosides (30s), Tetracyclins (30s) e.g. gentamicin, erythromicin -oxacin = inhibits DNA synthesis (by stopping DNA girase/topoisomerase from unwinding/uncoiling its bacterial DNA to replicate it) Quinolones "Sulf" or "meth" = inhibits folic acid synthesis

Q4. Dendritic cells in the skin mostly: 1) Are MHC class II+ 2) Present antigens to B-cells 3) Lack CD80 and CD86 4) Are derived from neutrophils 5) Express immunoglobulin at the cell membrane

1) Are MHC class II+

Q4. Dendritic cells in the skin mostly: 1) Are MHC class II+ 2) Present antigens to B-cells - are Dendrititic cells antigen presenting? 3) Lack CD80 and CD86 4) Are derived from neutrophils 5) Express immunoglobulin at the cell membrane

1) Are MHC class II+ *follicular dendritic cells present antigen to B cells! *defo have CD80, CD86 + lots of other costimulatory molecules *only B cells express ig on membrane!

3 persistent viruses?

1) Herpesviruses e.g. varicella-zoster virus (VZV) 2) HIV 3) HBV, hepatitis C virus

How does the host respond against GAS?

1. The 1st stage of defence is the epithelial cells. • These are an effective barrier against GAS • Autophagosomes envelope GAS and kill GAS when they fuse with lysosomes • Thus, autophagy may be a mechanism by which nonphagocytic epithelial cells protect themselves against GAS invasion. 2. The 2nd stage of defence is by host phagocytic cells: The host responds by an acute INFLAMMATORY response. Streptococci are OPSONIZED by activation of the ALTERNATIVE and LECTIN innate complement pathways (and the classical pathway in the presence of anti-M protein antibodies in the plasma and tissue fluid). Neutrophils kill the pathogens by phagocytosis (helped by opsonisation).

1. How is diversity in the antibody's variable region created? 2. How many diff sets of these gene segments are there?

1. "Gene recombination": The variable region is encoded by 3 (for heavy chains V,D,J) and 2 (for light chains V,J) GENE SEGMENTS ! 40 diff V gene segments possible for heavy chain, as well as 27 diff D segments, and 6 J. Add this to the fact that you have diff variations on the gene segments in both different light chains -- & you have so much diversity in the antibody variable region! --> to allow you to recognise a variety of antigens :) 2. 3 separate sets (1 for heavy chains, and 1 for each light chain).

Which single one of the following is the most likely characteristic feature of primary HIV infection? 1. A glandular fever-like illness and/or encephalitis 2. Cerebral toxoplasmosis 3. Fever, rash & granulomas 4. Lymphoma 5. Progressive multifocal leucoencephalopathy

1. A glandular fever-like illness and/or encephalitis (if glandular fever is not due to Epstein-Barr virus then you should be thinking HIV) - encephalitis - inflammation of the brain - seizure, fever, reduced level of consciousness *cerebral toxoplasmosis - happens in AIDS *no granulomas - that's TB *Lymphoma - can get in AIDS - but most likely caused by Epstein Barr virus *progressive multifocal leucoencephalopathy - damage to white matter of brain - due to JC virus - opportunistic infection seen in AIDS

Q1 The antiviral agent acyclovir (ACV) is best described as: 1. A nucleoside analogue 2. Being active against HIV 3. An antibiotic 4. Being phosphorylated by cellular thymidine kinase 5. ACV treatment can eliminate latency in herpesvirus infections

1. A nucleoside analogue *2. Herpes simplex (encephalitis) & Varicella Zoster (shingles) *4. No - 1st step to activate it so it can be further phosphorylated & inhibit a polymerase is done by a VIRAL kinase, not a cellular kinase (that's why it's not toxic in humans) *5. No cos the latent form is in plasmids in cells so can't be eliminated

1. Eisinophils have granules that stain with what kind of dyes? & which pathogens do they defend against? 2. Basophils are similar to what other type of cell?

1. Acid dyes. Parasites. 2. Mast cells.

What are 3 methods of indirect transmission from an infected person/animal?

1. Airborne: By inhaling air containing virus in aerosols or droplet nuclei or virus in shed skin, dust etc. 2. Vehicle-borne: via any substance serving as an intermediate means by which an virus (or any infectious agent ) is transported and introduced into a susceptible host through a suitable portal of entry e.g. Via fomites e.g. respiratory secretions, smallpox; Via the faecal/oral route; via needlestick/blood transfusion/organ transplant 3. Vector-borne: via the bite of an arthropod a.k.a. vector (e.g. mosquito, tick) which has previously bitten an infected person/animal.

Which 3 groups of antibiotics inhibit protein synthesis by binding to the ribosome?

1. Aminoglycosides (streptomycin) 2. Macrolides (erithromycin) 3. Tetracyclines

Name the 5 main stages of viral replication

1. Attachment 2. Penetration/virus entry 3. Uncoating (viral genome released from protein coat/capsid) 4. Expression & replication of viral genomes 5. Viral assembly & release https://www.youtube.com/watch?v=0-PCxrtLicg

2 types of autoimmune disease?

1. Autoantibody mediated 2. T cell mediated e.g. MS

4 types of transplantation:

1. Autograft: Donor & recipient are genetically identical - transplant from 1 part of the body to another 2. Isograft: between genetically identical individuals e.g. monozygotic twins 3. Allograft (allogeneic: people of the same species but w a diff genetic makeup) 4. Xenograft: from 1 animal to another animal of a different species

1. Although both B-lactams & glycopeptides act on the cell wall, how does their antibiotic mechanism differ? 2. Why is this difference important?

1. B-lactams inhibit ENZYMES which synthesize peptidoglycan --> cell lysis. GLycopeptides don't act on enzymes, but rather, physically obstruct the cell wall from coming together! 2. Use glycopeptides as treatment for MRSA! Because MRSA has mutated enzymes involved in cell wall synthesis...so B-lactams won't be able to inhibit these enzymes.

Which 2 nutrients do commensal bacteria supply to the host body? :)

1. B12 (gut flora produces B12) 2. Oral flora releases hydrogen peroxide used in salivary peroxidase antimicrobial system - inhibits coliforms

Which 2 organs have decreased output of naive cells as we age? Does this mean overall numbers have reduced though?

1. Bone marrow (fewer progenitor cells produced ---> so fewer specialized immune cells that develop from the progenitor cells) 2. Thymus (decreased output of naive T cells) NO - the overall level of T cells remains constant throughout life! It's just the no of naive T cells that declines... --> the naive T cells are rapidly converted into memory T cells for antigens that are continuously present e.g. Cytomegalovirus & Epstein-Barr Virus. We get LOADS of clones of these memory T cells, but very few memory cells for more rare antigens e.g. Influenza & VZW, making it hard for the immune system to respond to new antigens & mount an effective response against a vaccine

1. What is the most convenient method to measure virus titre (concentration)? 2. What is a plaque?

1. By plaque assay using tissue culture. This is less time-consuming than counting pocks on the chorioallantoic membrane of infected eggs. 2. Plaque: area of cell destruction that is visible in the tissue culture monolayer

1. What is the part of the antibody variable region that specifically recognises the antigen? 2. How many .....s are there?

1. CDRs (Complementarity determining regions) 2. 3 on each heavy chain and 3 on each light chain.

Possible therapies to rejuvenate the immune systems of the elderly?

1. Calorie restriction (not enough evidence + mice exp showed the anorexic ones acc died more cos didn't have enough energy to combat influenza) Buttt it is supposed to increase no of naive T cells & T cell diversity 2. Cytokine and hormone treatment : Growth hormone & keratinocyte growth factor to enlarge the thymus 3. Prevent CMV Although no vaccination available & the drug Ganciclovir is v toxic (carcinogenic + many side effects) 4. Nutrition: high fibre & post-biotics to promote healthy microbiota & decrease inflammation 5. Exercise: Regular exercise is associated with improved vaccination responses in older adults, better naïve/ memory- senescence cell ratios, macrophage and neutrophil responses and ab titres. 6. Weight loss: adipose tissue contributes to the low-grade inflammation (As well as secreting many inflammatory mediators such as TNFα and Il-6, the levels of anti-inflammatory cytokines are reduced in fatty tissue.) 7. Managing stress (also contribs to low grade inflammation) 8. Drugs which interfere with signaling pathways involved in SASP (Rapamicyn, inhibitor of mTOR and Metformin (normally used in diabetes treatment) 9. Removal of senescent cells (potensh) - Animal studies have shown that removal of senescent cells can delay age-related degenerative pathologies 10. Plasma transfusion from young to old ppl (potensh)

Describe 3 ways microbes avoid phagocytosis

1. Capsule inhibits contact 2. Toxins kill phagocyte 3. Opsonizing IgG neutralised e.g. Staph secretes protein A which neutralises antibody IgG

How bacteria can avoid complement?

1. Cleavage: Bacteria can secrete protease e.g. C5ase which destroys complement component C5a! e.g. H. influenzae 2. Distant binding site: If complement is activated away from the cell surface, it cannot damage the cell. So some bacteria ensure complement only gets activate far away from the cell e.g. Salmonella 3+4 apply to ENCAPSULATED bacteria only 3. Inhibit binding of complement to capsule e.g. Staphylococcus 4. Inhibit insertion of the MAC e.g. Neisseria

3 ways to avoid ADAPTIVE immunity (recognition by lymphocytes)

1. Concealment e.g. hide in areas of the body which aren't normally visited by lymphocytes (skin, excretory glands, gut contents, brain, testis, placenta = "immunologically privileged" areas) or e.g. mimic host antigens --> but this often leads to autoimmunity rather than pathogen survival or e.g. Cytomegalovirus is rly good at intrerfering w antigen processing and/or presentation so viral peptides don't get displayed to cytotoxic T cells! 2. Antigenic variation e.g. African trypanosome: over 1000 diff variable surface glycoproteins (VSGs) can be switched on and off - host needs to make new antibody every week + can't keep up 3. Immunosuppression e.g. HIV suppresses T Cell immunity

1. The Toll-like receptor 9 (TLR9) pattern recognition receptor recognizes: 2. TLRI/TLR2 heterodimerrecognizes: 3. TLR2/TLR6 heterodimer 4. TLR4

1. CpG motifs (microbial dinucleotide sequences) 2. bacterial lipopeptides 3. Lipoteichoic acid (gram positive bacteria) 4. LPS in gram neg bac

1. How does DNA polymerase synthesise a new strand of DNA? 2. How do nucleoside analogues work? 3. Which 2 enzymes are acting in the process before Acyclovir inhibits the viral DNA polymerase? And, subsequently, how can the Acyclovir virus become resistant?

1. DNA polymerase synthesises a new strand of DNA by extending the 3' end of an existing nucleotide chain, adding new nucleotides matched to the template strand one at a time to the 3' hydroxyl of the deoxyribose via the creation of phosphodiester bonds. When a nucleotide is added to a growing DNA strand, two of the phosphates (pyrophosphate) are removed and a phosphodiester bond is created so that the nucleotide is attached to the growing chain. 2. Aciclovir triphosphate acts as both inhibitor and substrate of the viral DNA polymerase, competing with GTP and being incorporated into DNA, leading to chain termination because aciclovir lacks the 3'-hydroxyl group required for chain elongation. 3. First the viral kinase, then cellular kinases. Acyclovir resistant virus strains occur if there is a mutation in the herpesvirus thymidine kinase or herpesvirus DNA polymerase.

How can some viruses traverse the basement membrane of the epithelium & become disseminated/systemic?

1. Enter an M cell: lymphocytes & antigens can then travel thru the bm 2. Viruses cause inflammation via cytokines ---> Inflammation increases the permeability of the basement membrane! (this is meant to only allow immune cells to get to come thru but unfortunately viruses slide thru too)

1. What characterises the localised immune response to TB? 2. Why does the body generate a "g..."?

1. Formation of a granuloma: wall of macrophages surrounding an area of necrosis 2. It can't kill the TB (knows it evades phagocytosis), so it thinks, ok then I'll contain it! Builds a structure around it (a granuloma) --> It's ultimately the IMMUNE RESPONSE which contains the damage but also causes the main symptoms! ---> inflammation ---> But also -- if there is an airway connecting to the granuloma, the TB can spread to the rest of the body through here

1. Drug used for CMV infection? 2. Besides Acyclovir, which other drugs are used for the Herpes virus?

1. Gangcyclovir 2. Cidofovir, Foscarnet

Main 3 groups of microbes that produce antibiotics?

1. Gram-positive rods 2. Funghi (e.g. Penicillium notatum: Penicillin) 3. Actinomycetes

Acyclovir 1. What is Acyclovir used to treat?

1. Herpes Simplex, Varicella Zoster virus

3 phases & mechanisms of transplant rejection?

1. Hyperacute - immediate - due to pre-existing antibodies - type 2 hypersensitivity 2. Acute - adaptive immunity - activation of lymphocytes (which is why it takes a few days) - type 4 hypersensitivity (T cell mediated) 3. Chronic - multiple mechanisms, but all organs seem to get rejected as many years pass

4 types of hypersensitivity reactions (1-3 antibody-dependent; 4 is T cell dependent)

1. IgE mediated mast cell degranulation (inflammation) e.g. hayfever, asthma 2. Antibody to cell surface antigens (cell death - by either phagocytes, NK cells or activates complement pathway & generation of MAC) e.g. Goodpasture's syndrome , haemolytic disese of the newborn 3. Immune complex mediated (complexes get deposited e.g. in kidney & trigger activation of complement) e.g. rheumatoid athritis, systemic lupus erythematosus (SLE) 4. Delayed type, T- cell mediated (antigen activates Th1 T cells which release cytokines like IFN which stimulate macrophages to release TNF etc --> inflammation) e.g. contact dermatitis, tuberculin skin reaction (Mantoux test) 5. (newly added) -- stimulatory e.g. Grave's disease (antibody mimics the effect of the receptor ligand and stimulates the thyroid gland to produce hormone)

Kaposi's sarcoma 1. Involves the internal organs in HIV patients 2. Is a common disease worldwide 3. Is caused by an RNA virus known as Kaposi's sarcoma herpesvirus 4. Is irreversible in immunosuppressed organ transplant patients 5. Was first described in elderly African men in the 1980's

1. Involves the internal organs in HIV patients (just on skin in non-HIV patients) * Is caused by Herpes virus which is a DNA virus! KSHV

3 types of vaccine?

1. Live/attenuated: don't cause disease e.g. MMR 2. Killed/inactivated : chemically inactivated form of the agent (but antigens r still present) e.g. flu 3. Sub-unit: synthetic isolated component of the antigen - made using recombinant DNA technology (not giving the person the natural antigen) e.g. Hep B + Conjugate: protects against polysaccharide antigens by combining them w a protein carrier so they are more easily recognised by T & B cells!

2 examples of successful prophylactic cancer vaccines:

1. Liver cancer: HBV vaccine 2. Cervical cancer: HPV vaccines (Gardasil and Cervarix)

4 assumptions that need to be satisfied for a virus to be eradicated?

1. No asymptomatic carriers still casually living w it 2. Immunisation stops transmission (unlike the inactivated 1955 polio virus) 3. No non-human host/reservoir (can't be having animals still harbouring it) 4. Limited virus stability (virus must have minimal stability in the environment)

Tissue culture: What does the medium you let the cells grow in in a plastic flask need to contain? (3)

1. Nutrients 2. Antibiotics 3. pH indicator

1. After a point-of-care test for RSV antigen (rapid immunochromatographic assay designed to detect RSV fusion protein in nasal washings or nasopharyngeal (nose and throat) swab specimens), what further tests might you want to perform? 2. Special step needed for PCR of the RSV virus (note it is a RNA virus)?

1. PCR & Light microscope of tissue culture The result requires confirmation by an alternative method such as RT-PCR or, less commonly, by immunofluorescence or viral culture. The specificity of rapid POCTs such as this may not be as high as that of traditional laboratory tests; false-positive results may occur (specificity range 83%-96%). 2. Reverse transcription first to make it DNA! RSV has an RNA genome and PCR can only amplify DNA. Therefore, the RNA has to be reverse transcribed into DNA (i.e complementary DNA or cDNA) by reverse transcriptase before PCR amplification

2 ways to evade the immune response?

1. Passive "hide" 2. Active "change"

Dendritic cells 1. Main function? 2. Therefore what molecules do they need?

1. Present antigens to T cells 2. Both MHC class 1 and class 2

Invasive candidisis - fatal ! 1. Causes? 2. Who is at risk? 3. How to diagnose?

1. Primarily endogenous source - GI tract • Overgrowth through broad spectrum antimicrobial use • Loss of integrity of the gut Mucosa - chemotherapyor surgery Exogenous sources • Vaginal tract during delivery • IV drug users • Hands of health care workers - ICU/PICU/Burns 2. a. patients w GI surgery (break down in mucosal barrier & GI is where the candida lives) b. HIV c. tumours d. v old & v young 3. ONLY way to diagnose is by blood cultures! So if patient only has organ involvement then radiology is only way to look for lesions -- so v difficult to diagnose--> high mortality

1. How do quinolone antibiotics work? --Name the 2 enzymes they affect 2. Safety & effectivity?

1. Quinolones (e.g. ciprofloxacin) inhibit 2 enzymes which supercoil bacterial DNA (DNA girase & DNA troposiomerase) (interfere with DNA replication & transcription) 2. Safe - low toxicity because humans don't have these enzymes BUT these enzymes are easily mutated! So high resistance

2 ways HIV is persistent & evades immune response?

1. Rapid genetic change 2. HIV envelope (Env) is protected by a glycan shield

Central tolerance: B cell selection Survival requires generation of a function antigen receptor If an immature B cell binds a self-antigen in bone marrow, what 2 things could happen to it?

1. Receptor editing (change specificity) - downregulate that receptor and put a new one on the surface 2. Deletion (negative selection)

2 differences between BCR antibody and soluble antibody?

1. Soluble antibody lacks the transmembrane and cytoplasmic sequences 2. Soluble antibody now has IgM in pentamers and IgA dimers (as opposed to the monomers in BCR), as well as additional proteins associated with the antibody's polymerisation & transport

Polio 1. Transmission? 2. How does it spread to other parts of the body? 3. When was the Salk inactivated vaccine introduced?

1. Spread by ingestion & then virus replicates locally in the GI tract 2. Transit via blood (viremia) to other tissues **dang if motor neurons (kills motor neurons --> paralysis) 3. 1955 (but only prevents u from getting the disease - doesn't ERADICATE it ) --> 1963 live attenuated vaccine -- can stop transmission & eradicate it

How to avoid acute rejection of a transplant? 3 drugs which block the T-cell activating pathway

1. Tacrolimus 2. Cyclosporine 3. Rapamycine *ensure you give as little as possible to create enough immunosuppression to prevent rejection but not completely suppress the immune system

Why is RAG / recombinase enzyme joining of V-D-J segments IMPRECISE? (2 parts of process)

1. deletion-- RAG complex removes base pairs 2. addition - then joins them back together again -this increases genetic diversity, but allows for many errors to be made e.g. Rec enzyme needs to make sure it is deleting an exact multiple of 3 bp... but at the end of the day, these inaccuracies increase diversity

1. What do Addressin molecules do? 2. Where are they? 3. What about homing receptors?

1. These Addressin molecules assure skin lymphocytes go back to skin, gut lymphocytes go back to gut etc 2. Addressin molecules are on the relevant blood vessel endothelium 3. Homing receptors on the surface of lymphocytes also help to guide them back to the right mucosal site.

1. What is the main role of IgA in the gut? 2. What transports the IgA across the mucosal barrier? (from the basal surface to the mucosal surface)

1. To stop bacteria or viruses in the gut from sticking to the mucosal surface & penetrating the gut (but remember, ppl with IgA deficiency survive, so this is not sooo imp.) 2. The poly Ig receptor

What are some safety concerns with antibiotics? (5)

1. Toxicity: --kidney & ear damage (aminoglycosides) --liver damage (drug used for TB - sometimes has to be stopped -- pseudomembranous colitis - c. difficile overgrows if u kill off normal commensal gut flora & starts releasing toxins which can damage the gut 2. Interactions w other medications -- can neutralize effectiveness of contraceptive pill 3. Allergic reactions e.g. to B-lactam ring 4. Foetal damage/ risk to pregnant women --Tetracycline --> can discolour children's teeth & cause liver damage in pregnant women --Floroquinolones may cause cartilage damage 5. Antibiotic resistance

Cancer is caused by genentic damage in oncogenes & TSG. What can cause these mutations?

1. Viruses (HBV - liver, KSHV, EBV, Papillomaviruses 16&18) - viral infection is necessary but not sufficient to cause cancer 2. Environmental carcinogens 3. Random mutations

Which anti-cytokine drugs can treat 1. rheumatoid athritis & 2. psoriasis?

1. anti-TNF 2. IL-17 (antibodies against it)

Role of lymphoid tissue in tissue immunity (3)

1. antigen trapping 2. cell-cell communication 3. separate sites of B (cortex) & T cell (paracortex) activation

How do some microbes avoid death IN the phagocyte (they haven't managed to avoid being phagocytised) (3)? + examples

1. escape from phagosome e.g. Leishmania & Trypanosoma cruzi 2. inhibit lysosome & phagosome fusion e.g. M.TB, toxoplasma 3. remain in phagosome, allow it to fuse with lysosome, but resist attack from the oxygen radicals and the lysosome's other enzymatic contents e.g. Staphylococci + M.TB

1. What is a keloid/hypertrophic scar? 2. Exuberant granulaton tissue (proud flesh)?

1. excessive collagen leads to 'tumour-like' outgrowth (genetic predisposition -black > white) 2. formation of excess granulation tissue which blocks regeneration of epithelium

3 factors that contribute to the development of autoimmune disease? (need multiple)

1. genetic susceptibility 2. environmental influences 3. failed regulation

2 key changes to the gut in old people that affect the immune system?

1. gut microbiota change due to increased serum levels of pro-inflammatory mediators such as IL-6 and IL-8 2. increased intestinal permeability (impaired intestinal epithelial barrier) ----> bacterial bacterial products have an increased chance of stimulating the innate immune cell

3 forms of protection against fungal infection?

1. immune system 2. mucosal surfaces 3. epithelial surfaces

Name 5 factors which impair healing

1. infection 2. early wound movement or constant disruption to healing process 3. foreign material (cannot be phagocytosed) 4. poor vascularisation e.g. atherosclerosis & other causes of vascular stenosis reduce vessel flow 5. Glucocorticoid (steroid drugs) (their anti-inflammatory action impairs collagen synthesis) 6. Immune suppression (reduced or poorly functioning leucocytes) 7. Diabetes (increased risk of infection & leads to poorer overall cell function) 8. Nutrition (protein, vit c & zinc espesh imp)

3 ways to avoid cytokines + examples for each

1. inhibit cytokine production e.g. Hep B inhibits IFNa production 2. Inhibit cytokine RECEPTOR EXPRESSION e.g. Trypansoma cruzi downregulates IL-2R expression 3. Produce a homologue of an immunosuppressive cytokine e.g. Epstein-Barr virus has a gene encoding 'viral IL-10'

2 causes of immunosenescence (deterioration of immune system as we age)

1. loss of lymphocyte function/ repertoire diversity (homeostatic expansion) 2. chronic inflammation (senescent cells, activated monocytes/macrophages)

1. Remember the epidermis is avascular...so what type of microbes can live here? 2. But what type of bacteria might you find on the surface of the skin?

1. obligate ANaerobes 2. aerobes (e.g. coagulase-negative staphylococci (CNS), diphtheroids, Acinetobacter) are found on the surface of the skin

What 2 things do we monitor to test antiviral efficacy? -- How would you determine what genes are causing the resistance?

1. plasma HIV RNA load 2. CD4 lymphocyte counts. *Rising virus load and falling CD4 counts in the face of therapy indicates resistance -- Plasma viral RNA sequences determines the genomic basis for resistance ---Resistance genotyping is routine to allow optimal drug choice

1. How does Legionella pneumophila survive phagocytosis? 2. What is the Dot/icm system?

1. survives phagocytosis by blockingck phagosome-lysosome fusion and inhibit innate immune defences - stops lysosome forming 2. Dot/icm system: defect in organelle trafficking/ intracellular multiplication

What do you add in each of the 3 stages of the ELISA test?

1. the antibody 2. enzyme that binds to that antibody 3. substration (which when bound to the antibody will cause a colour change so the presence of the antibody is easily identifiable!)

What happens in germinal centres?

1. the major site of T cell help 2. somatic hypermutation, affinity selection & isotype switching

2 diff forms of funghi:

1. yeast: single celled budding organisms, with/without hyphae 2. moulds: multicellular filamentous organisms with hyphae and fruiting structures + produce spores *but some funghi are dimorphic! Above 37c (once in host) switch to yeast e.g. histoplasma coccidioides

Each antibody unit has 2 arms and therefore how many Fabs?

2 Fabs. Each Fab contains the V domain & 1st C domain of 1 of the heavy chains, as well as all V and C domains of 1 of the light chains.

Which is a requirement for virus eradication by immunisation? 1. Asymptomatic carriage 2. Immunisation stops transmission 3. Non-human reservoir 4. Virus stability in the environment 5. Insect vector

2. Immunisation stops transmission (think of Rzero)

Q2. The best description relating to emerging infections and disease is that: 1. Influenza virus H5N1 is the result of genetic reassortment between pig & human influenza viruses 2. Emergence requires infection and transmission 3. Influenza pandemics occur because of antigenic drift 4. Legionnaire's disease is transmitted by ticks 5. Most emerging infections are drug resistant

2.Emergence requires infection and transmission *H5N1 = bird flu *Legionnaire's grows in water *5: some are e.g. supergonorrhoea + multi drug-resistant TB but not all are

What is the approximate number of D segment genes in the immunoglobulin heavy chain gene locus? A 40 B 23 C 9 D 6 E 1

23 D genes!! The correct answer is B. Note that D regions are only found in the heavy chain; the light chain gene loci lack D segments. The approximate number of immunoglobulin heavy chain V genes is 40. There are 9 genes encoding immunoglobulin heavy chain constant regions which specify the class and subclass of antibody: IgG1-4, IgA1-2, IgM, IgD, IgE. There are 6 immunoglobulin heavy chain J genes. Multiple D genes are present. Different D genes are used by different B-cells, contributing to the generation of antibody diversity.

Q5. The antigen receptor on a T-cell is best described as: 1) Being called CD3 2) A homodimer 3) Unique to T-cells 4) Only recognizes peptides presented by MHC class I 5) Associated with CD80 and CD86 ?

3) Unique to T-cells (TCR - other cells like B-cells have antibodies as receptors) 1) *CD3 molecules are costimulatory molecules which ASSOCIATE with the antigen receptor - but they are not the antigen receptor themselves 2) *a heterodimer (both an alpha and beta chain - 2 chain) 4) recognizes peptides presented by MHC class I (or 2!!) - depends on what kind of T cell you are 5) CD80 + CD86 = also costimulatory molecules required for T cell activation but not associated w the antigen receptor per say

Which of the following statements is true about hepatitis B virus?

3. Core antigen forms a protective nucleocapsid around the genome YESS 1. It is an RNA virus (NO it's a DNA virus) 2. It has a segmented genome (No) 4. Surface antigens are only found in soluble form in blood (No) - surface antigens on the envelope too! And another antigen E antigen is in the blood too! 5. It encodes for an RNA- dependent RNA polymerase (No - it's DNA polymerase)

Which of the following is true about the diagnosis of hepatitis B? 1. Antibody to HBcAg is used to monitor the response to HBV vaccine 2. Both serum and stool samples are needed to diagnose HBV infection 3. HBsAg in a serum sample indicates hepatitis B infection 4. IgG anti-HBc indicates recent infection 5. Measurement of HBV RNA samples is used to monitor antiviral therapy

3. HBsAg in a serum sample indicates hepatitis B infection -- yes, presence of the antigen indicates infection *1. NO -use SURFACE antigen to measure vaccine response *2. no - only serum samples are used (but yes stool is used in Hep A) *4. - IgM shows acute/recent response *5. DNA- by PCR

Laboratory diagnosis of HIV infection 1. Cellular immunity to HIV is routinely used for diagnosis 2. Current tests for HIV antibody are prone to false positive results 3. Testing for HIV p24 antigen reduces the seronegative window period & provides earlier diagnosis than HIV antibody tests alone 4. HIV-specific IgG in the newborn baby's blood indicates HIV infection 5. HIV antibody tests generally become unreliable after the onset of immune deficiency In what way is the diagnosis of HIV infection usually made and what should you understand by the term confirmation in this context?

3. Testing for HIV p24 antigen reduces the seronegative window period & provides earlier diagnosis than HIV antibody tests alone In what way is the diagnosis of HIV infection usually made and what should you understand by the term confirmation in this context? Combined tests - serum anti-HIV and p24 antigen Tests must be confirmed by a qualified lab, as although they are specific, there are quite a few false positives. Confirmation is done by PCR of a second sample from the patient. Newborn babies need to have direct virus detection - anti-HIV antibodies may have come from the mother.

Antiretroviral therapy 1. Resistance genotyping involves culture of HIV in lymphocytes 2. Azidothymidine, a nucleoside analogue, inhibits the HIV integrase. 3. Use of HAART may lead to immune reconstitution 4. Monitoring the response to antiviral drugs is achieved by regular measurement of HIV DNA in plasma 5. HIV infection can sometimes be cured by the use of HAART

3. Use of HAART may lead to immune reconstitution --immune reconstitution: immune system is getting some of its power back - virus stops replicating so more CD4 cells now *Azidothymidine, a nucleoside analogue, inhibits RNA or DNA polymerases *is HIV RNA! DNA will just tell you if the person is infected; not if the virus is replicating

Treatment for patient with meningitis due to Neisseria meningitidis infection?

3rd generation cephalosporin e.g. Ceftriaxone or Cefotaxime. (Cephalosporin = a beta lactam)

An immune system can provide cancer immune surveillance & thus reduce the incidence of cancer 1. Immune surveillance works only against cancers caused by viral infection 2. Surveillance is mediated primarily by NK cells killing transformed cells 3. The immune system protects against cancer in childhood but not in adults 4. Escape from immune surveillance is a step in cancer development 5. Immuno-edited cancer variants are efficiently eliminated by the immune system

4. Escape from immune surveillance is a step in cancer development *1 . well against viruses but also other cancers *2. immune surveillance is predominantly carried out by CD8 cells actually *3. - no - everyone *5. opposite - cancer mutates to escape surveillance & divide

The immune system of the elderly often exhibits 1. Hypogammaglobulinaemia (low levels of antibody) 2. A lower proportion of memory T cells 3. An increase in neutrophil numbers 4. Impaired response to influenza vaccines 5. Impaired inflammatory responses

4. Impaired response to influenza vaccines *3. decrease

Acute graft-versus-host disease 1. Is a characteristic feature of liver transplantation 2. Can only occur if donor & host are mismatched at the HLA locus 3. Occurs when donor derived B cells produce antibodies 4. Often targets the host gastrointestinal tissue 5. Can be prevented by deleting donor T cells from the allograft

4. Often targets the host gastrointestinal tissue (gut, skin & lung (epithelial tissues) are v susceptible to GvH) *1. No - GvH only occurs when you transplant white blood cells, bone marrow, or stem cells (not solid organ) *2. Can also occur if donor & host match at HLA but not at diff sites *3. GvH is a T cell mediated response - T cells derived from the donor attack the cells of the host *4. Yes can improve outcome, but even if you deplete all the T cells, the new T cells made post-transplant from the stem cells of the donor in the host can also recognise the host as foreign - because the host thymus is repopulated w dendritic cells also coming from the donor - so now the donor cells in thymus are mismatched from the other cells in the body

Besides: •1. Multiple V, D, J minigenes • 2.Imprecise joining • 3. Independent light and heavy chains What else contributes to antibody diversity? N.B. this 1 occurs after exposure to the antigen + What is the purpose of these extra changes? + Which enzyme is involved?

4. Somatic hypermutation (SHM) Once antibody has encountered an antigen, it goes through somatic hypermutation to increase antibody-antigen AFFINITY + makes them bind better :) + enzyme: AID (activation induced deaminase

What are Baltimore groups 4 and 5?

4: Single stranded POSITIVE-sense RNA - the viral genome here can itself act as mRNA! 5: Double stranded NEGATIVE sense RNA - viral mRNA is transcribed directly from the viral genome e.g. influenza virus *remember mRNA is positive sense, so you already have negative sense RNA here as the template! :)

Immunodeficiencies: 1. With impaired T cell function are characterised by severe bacterial infections 2. With impaired T cell function can be due to a defective Bruton tyrosine kinase gene (btk)with 3. With impaired antibody production most frequently occur in the IgM class 4. With impaired neutrophil function are treated with pooled serum antibodies from normal individuals 5. Are common in cancer patients treated with cytotoxic drugs

5. Are common in cancer patients treated with cytotoxic drugs (Cytotoxic drugs tend to target cells where there is most proliferation --> Immune system & gut are the tissues w/ most proliferation -> so they are harmed most) *1. Severe bacterial infections are acc usually associated w severe B cell or antibody production (not T cell production) *2. Bruton tyrosine kinase gene (btk) is a kinase in B cells not T cells! *3. most common is IgG *4. No not treated w pooled serum cos that replaces the antibody & replacing the antibody won't help the neutrophils!

Q6. Which statement about hepatitis B vaccine is true? 1. It is a live attenuated vaccine. 2. One dose is enough to ensure an adequate immune response. 3. There is currently no effective vaccine available against hepatitis B virus. 4. Vaccine also provides protection against hepatitis E virus. 5. Response can be monitored by measuring antibody to hepatitis B surface antigen in the recipients serum.

5. Response can be monitored by measuring antibody to hepatitis B surface antigen in the recipients serum. *1. No it's inactivated *4. No, altho there is a A+B combo vaccine.

Q5. Which of the following statements about hepatitis B treatment is true? 1. Acute hepatitis B infection is always treated. 2. Hepatitis B immunoglobulin is used to treat HBV infection. 3. Hepatitis B is treated for a defined time period only. 4. Hepatitis B treatment usually includes interferon gamma. 5. Reverse transcriptase inhibitors (i.e. tenofovir) can be used to treat HBV infection.

5. Reverse transcriptase inhibitors (i.e. tenofovir) can be used to treat HBV infection. -- aim: *1. can resolve itself *2. can be used as prevention but not as treatment *3. rarely - often much longer treatment *4. used to include interferon beta but very rarely nowadays

How could "Reverse vaccinology" help protect against N. Meningitidis, even though the antigen fHBp has over 500 sequence variants? Also note they can't use a conjugate vaccine for N. Meningitidis because it's polysaccharide capsid is structurally similar to human sugars

500 sequence variants come under 3 groups. Scientist could engineer 1 recombinant antigen that encompasses all the epitopes

What are the immunological events leading to gout ?

= a form of inflammatory athritis Gout is caused by high blood levels of uric acid that crystallize and form painful deposits in the joints. Excessive amounts of alcohol, protein, heavy foods & coffee cause uric acid levels to rise. i) Innate hypersensitivity response to uric acid crystals ii) high concentrations of uric acid lead to formation of uric acid crystals Uric acid crystals activate the inflammasome leading to activation of caspase 1, and release of IL1 (especially beta) iii) IL1 and activates subsequent parts of innate immune response (inflammation), including leukocyte infiltration into joint, and release of inflammatory mediators. Iv) can prevent by treatment with IL1 inhibitors (e.g. anakinra, rilonacept and canakinumab).

What are "complement" molecules in the immune response?

A cascade system of about 20-30 different molecules. Especially effective against bacteria. (innate response)

What contributes to the low grade chronic inflammation in old age, coined "inflammaging"?

A decrease in IL-2 plasma levels alongside an increase in pro- inflammatory molecules, particularly TNFα, IL-6 and CRP. These circulating inflammatory parameters can be positively correlated with each other, suggesting a generalised activation of the entire inflammatory network. *Suffering from an inflammatory disease acts as a risk factor for other pathologies of old age, setting up a positive feedback loop whereby the systemic inflammatory response associated with diseases of old age initiates and propagates further pathologies

What is a "dead-end host" in the viral transmission cycle?

Any host organism from which a parasite cannot escape to continue its life cycle. Humans are dead-end hosts for trichinosis, because the larvae encysted in muscle and human flesh are unlikely to be a source of food for other animals susceptible to this parasite.

Acute inflammation can be initiated by: A Mast cell activation B Influx of neutrophils C An increase in vascular permeability D C3 E Lysozyme

A Mast cell activation Activation of mast cells releases chemotactic factors for neutrophils and also vasoactive mediators such as histamine. The influx of neutrophils represents part of the acute inflammatory response itself. The increase in vascular permeability is an essential feature of the acute inflammatory response itself.

Aedes

A genus of mosquito; a vector of the viruses: Dengue Fever, Yellow Fever, the Zika Virus .

Ebola

A viral haemorrhagic fever Viral hemorrhagic fevers are infectious diseases that interfere with the blood's ability to CLOT. These diseases can also damage the walls of tiny blood vessels, making them LEAKY. The INTERNAL BLEEDING that results can range from relatively minor to life-threatening. Some viral hemorrhagic fevers include: Dengue Ebola Lassa Marburg Yellow fever

NK cells lack receptors with: A Specificity for MHC class II molecules B Specificity for MHC class I molecules C C-type lectin domains D Immunoglobulin-like domains E Specificity for the Fc region of IgG

A. NK cells do not bear receptors with specificity for MHC class II, only class I. NK cells have inhibitory receptors for MHC class I molecules, thereby preventing them killing normal cells which usually express MHC class I. Cells that lack MHC class I because they are malignant or infected are not able to engage the inhibitory NK receptors and are therefore killed. Examples of both activating and inhibitory NK receptors can be found among the C-type lectin domain receptors on NK cells. Examples of both activating and inhibitory NK receptors can be found among the killer immunoglobulin-like receptors (KIRs) on NK cells. NK cells utilize Fcγ receptors to mediate antibody-dependent cellular cytotoxicity of IgG-coated target cells.

A. What are the 2 major types of trypansomes (parasite)? B. And why are trypansomes hard to treat?

A. 1. Trypanosoma cruzi - Africa - vector: tsetse fly - AKA Sleeping sickness 2. Trypansome brucei - S. America - build up of balls of cytoplasm in muscle --> effect nerve fibers and heart function --> heart responds by building up more muscle & pumping harder --> enlarged heart & chronic heart disease "Chagas disease" B. Trypansomes are eukaryotes, so their cells are v similar to human cells --> so drugs which kill them would kill us too --> narrow therapeutic window. Often treated w arsenic-based compounds but dose must be managed v carefully.

A 10-month- old boy has recurrent bacterial infections and is diagnosed with a primary immunodeficiency disorder affecting B lymphocytes. Which is the enzyme most closely associated with both somatic hypermutation and class switch recombination? A. Activation-induced cytidine deaminase B. Cathepsin C. Granzyme B D. Recombination activating gene product E. Terminal deoxynucleotidyl transferase

A. Activation-induced cytidine deaminase (AID) AID is currently thought to be the master regulator of secondary antibody diversification. It is involved in the initiation of three separate immunoglobulin (Ig) diversification processes: Somatic hypermutation (SHM), in which the antibody genes are minimally mutated to generate a library of antibody variants, some of which with higher affinity for a particular antigen and any of its close variants Class switch recombination (CSR), in which B cells change their expression from IgM to IgG or other immune types Gene conversion (GC) a process that causes mutations in antibody genes of chickens, pigs and some other vertebrates.

A. Where are B cells mainly found in the lymph node? B. And T cells?

A. B cells recognise antigen in the CORTEX (within the germinal centre) B. T cells in the PARAcortex

What can filarial worms cause?

A. Lymphatic disease - we see swelling of skin & oedema when lymphatic vessels become blocked --> elephantitis B. Blindness Larva needs to pass through insect vector.

A. What are interferons? B. Which infectious agents are sensitive to them?

A. cytokine proteins which are produced in response to viral infections. They can stop the growth of viruses in cells. (innate response)

What can you give to babies born to infected mothers and after accidental exposure?

Active-passive immunisation - Hepatitis B immunoglobulin (HBIG) is prepared from donors with high anti-HBs titres

Levels of which molecule made in the liver are a good indicator of a patient's inflammation?

Acute Phase Proteins e.g. C-Reactive Protein (CRP) - helps phagocytes engulf bacteria! (innate response)

In a "one step growth curve" how do you ensure that the virus replication cycle is synchronised?

Add enough virus to ensure all cells become infected simultaneously!

Pneumocystis pneumonia Opportunistic pathogens -immunocompromised

Affects ppl w AIDS a lot too ! (like cryptococcus) - pulmonary infection Easy to diagnose: 1. Grocott's Methenamine Silver (GMS) stain SILVER STAIN 2. Immunofluorescence using monoclonal antibodies 3. X ray: see bilateral opacification Caused by lack of CD4 cells fighting the pathogen, so loads of CD8 cells are recruited instead + inflammation --> leads to direct epithelial damage, respiratory failure

Between CD4 and CD8 cells, which age more?

Age related changes occur to both CD4+ and CD8+ T cells, however the rate at which these changes are more pronounced on CD8+ T cells due to a greater homeostatic stability of CD4+ T cells.

What allows ageing cells to be recognised & phagocytosed? (Self-recognition - not good)

Gradual damage to carbohydrates on cell surfaces e.g. rbcs allow the cells to be recognised & phagocytosed.

Amantadine a. what does it treat? b. which stage of viral replication does it target?

Amantadine blocks the M2 protein ion transport channel; (normally protons entering this M2 channel allow the UNCOATING of the viral nucleic acid: so no protons = no uncoating) a. Influenza b. Inhibits virus uncoating and penetration into the host cell: Influenza haemagglutinin (virus attachment protein) binds to sialic acid on the cell surface (cell receptor) and the virus is taken up via vesicles (endosomes). In the acid environment of the endosome, protons enter the virus via the M2 protein ion transport channel allowing uncoating of viral nucleic acid. Amantadine blocks the M2 protein ion transport channel and inhibits uncoating.

How does Influenza evade the immune response? What des h.... do?

Antigenic SHIFT - reassortment of genome segments in haemagglutanin envelope protein Haemagglutanin binds to sialic acid on host cell FUSES membranes @ low pH in endosome

Neutrophils in Cerebrospinal fluid (CSF) - what does this indicate?

An infection! Neutrophils should not normally be in CSF.

Which bacteria live in sweat glands, and what do they release?

Anaerobic bacteria such as Proprionibacterium acnes live in sweat glands and release free fatty acids from sebum which inhibit other organisms

Which genus of mosquito can transmit malaria? What is the name of the most dangerous malaria parasite to humans?

Anopheles Plasmodium falciparum

Why are antibodies only protective against organisms that live outside cells (extracellular)?

Antibodies are unable to penetrate living cells

How do antifolate antibiotics work?

Antifolates (e.g. sulphonamides & trimethoprim) - inhibit synthesis of folic acid (needed to make pyramidines & purines) -- an essential metabolite - competitive inhibition of enzymes

Animal reservoir/ natural vertebrate host

Any person, animal, plant, soil or substance in which an infectious agent normally lives and multiplies. The reservoir typically harbors the infectious agent without injury to itself and serves as a source from which other individuals can be infected. The infectious agent primarily depends on the reservoir for its survival. It is from the reservoir that the infectious substance is transmitted to a human or another susceptible host.

Variant Creuzfeldt-Jakob Disease (human form of mad cow disease)

Arose as a zoonosis due to the prion of bovine spongiform encephalopathy being ingested by humans in beef Hard to treat: prions aren't destroyed by the extremes of heat and radiation used to kill bacteria and viruses, and antibiotics or antiviral medicines have no effect on them.

Which yeast infection is associated with neutropenia?

Aspergillus fumigatus

Autoantibodies: mechanisms of action (3) Give an example of each What is an opsonin? In autoimmune cytopenias, the autoantibodies act as opsonins -name 3 of these

Autoantibodies can be blocking (antagonist), stimulating (agonist) or opsonizing 1. Myasthenia gravis antibody blocks Ach from binding to its receptor 2. Grave's disease (type V) stimulates the TSH receptor Opsonin: An opsonin (from the Greek opsōneîn, to prepare for eating) is any molecule that enhances phagocytosis by marking an antigen for an immune response or marking dead cells for recycling Autoimmune cytopenias: - Autoimmune thrombocytopenia (platelets) - Autoimmune haemolytic anaemia (erythrocytes) - Autoimmune neutropenia (neutrophils)

AIRE autoimmune

Autoimmune Regulator Transcription Factor AIRE drives negative selection of self-recognizing T cells. When AIRE is defective, T cells that recognize antigens normally produced by the body can exit the thymus and enter circulation. This can result in a variety of autoimmune diseases.

Several features are necessary for effective vaccines, one of which is the ability to stimulate the generation of specific memory helper T cells. This requires the vaccine to contain antigen that can be processed and presented to the T cell receptor (TCR) on these cells. Which one of the following best describes the entire intact molecule that presents antigen-derived peptides to the TCR on human T cells? A) Class II alpha-chain. B) HLA. C) Leukocyte surface antigen. D) MHC. E) Peptide-presenting molecule.

B) HLA.

What are the molecules mediating signal transduction following antigen binding to cell surface immunoglobulin on a B-cell called? A Ig Fc B Ig-α and Ig-β C MHC D CD4 E CD8

B. Ig-α and Ig-β possess C-terminal cytoplasmic regions which become phosphorylated upon cross-linking of membrane immunoglobulin, leading to a rapid mobilization of intracellular calcium. The Fc part of a cell surface antibody molecule contains a short cytoplasmic tail but this is not responsible for signal transduction. MHC molecules are specialized for the presentation of antigenic peptides to the TCR. CD4 molecules are found on a subpopulation of T-cells, but not on B-cells, and they recognize MHC class II molecules. CD8 molecules are found on a subpopulation of T-cells, but not on B-cells, and they recognize MHC class I molecules.

A B-cell is able to make cell-surface and secreted versions of antibody using: A Different gene pools B Differential splicing C Different heavy chain class but the same light chain D Different light chain class but the same heavy chain E F(ab′)2 fragments *What is "receptor editing"?

B. Differential (alternative) SPLICING of a primary RNA transcript can produce antibody either with or without exons encoding a hydrophobic transmembrane sequence which leads to retention of antibody in the cell surface membrane! The gene pools used for the cell-surface and secreted antibodies are identical. Note that during early B-cell development if the antibody produced reacts against "self," the B-cell may be able to express a different light chain and be given the opportunity of avoiding tolerance induction mechanisms if the new heavy : light combination no longer reacts with self, a phenomenon referred to as receptor editing.

Which of the following statements is TRUE of IgE? A IgE is abundant in saliva B IgE binds strongly to mast cells C IgE cannot bind to macrophages D IgE activates the complement cascade E IgE has an insignificant role in worm infestations

B. IgE binds strongly to mast cells The Fcϵ receptor on mast cells, FcϵRI, binds IgE very strongly and cross-linking by antigen leads to mast cell activation and initiation of an inflammatory response. IgE is at a very low concentration in the body fluids. IgA is predominant in saliva. The FcϵRIIb isoform of the low affinity FcϵRII (CD23) is present on several different cell types including macrophages. IgE antibodies have an important role in defense against worm infection.

Which one of the following describes a mechanism of bacterial pathogenesis? A. Diphtheria toxin inhibits DNA synthesis resulting in the death of host cells B. Staphylococcus aureus can cause food poisoning via enterotoxins C. Streptococcus pyogenes does not produce toxins D. The polysaccharide capsule of E.coli is an exotoxin E. Vibrio cholera produces an exotoxin that inhibits synapses in the spinal cord What does diptheria toxin do? - Which bacteria causes diptheria? - is the toxin encoded by the bacterial genome?

B. Staphylococcus aureus can cause food poisoning via enterotoxins Diphtheria toxin is an exotoxin secreted by Corynebacterium diphtheriae, the pathogenic bacterium that causes diphtheria. Unusually, the toxin gene is encoded by a bacteriophage (a virus that infects bacteria).[1] The toxin causes the disease in humans by gaining entry into the cell cytoplasm and inhibiting PROTEIN synthesis.[2]

What is a microaerophile? Give 2 examples - what shape are these 2?

Bacteria that only grows in reduced percentage of oxygen e.g. 15-17% Campylobacter Helicobacter ^both Gram-negative rods

1. Why is it a problem for the immune systemthat telomeres shorten as we age? 2. Which enzyme compensates for the loss of telomere length during cell division?

Because telomeres are essential for maintaining chromosomal integrity, cells with critically shortened telomeres cease division (senescence) and are prone to apoptosis. Thus, the length of telomeres serves not only as a record of cell division history but also as a limit for the number of divisions that a cell can undergo. ---> But T Cells need to be undergo cell divisions in response to antigenic challenge! 2. Telomerase During ageing T cells display shorter telomeres and less telomerase activity.

Difference between Acyclovir (herpesvirus + Varicella-Zoster) and AZT (HIV) in terms of toxicity? What is the difference between nucleoside analogues and non-nucleoside analogues?

Both are nucleoside analogues. However, acyclovir requires a herpes virus thymidine kinase to become activated, where as AZT (azidothymidine) is solely activated by cellular kinases (not viral kinases), so acyclovir will not get activated in uninfected cells, whereas AZT will. Any cell is at risk of taking AZT up and having their DNA replication inhibited. This makes AZT more toxic. Nucleoside analogues require activation via intracellular phosphorylation whereas NNRTIs do not (e.g. Nevirapine)

Treatment for Hep B & C?

Both: Interferons (synthetic version of the cytokine; Interferon helps the body distinguish between cells infected by the virus and non-infected cells, targeting infected cells for destruction) Hep B: nucleoside/nucleotide RT inhibitors. Hep C: interferon in combination with ribavirin + new: viral protease and RNA polymerase inhibitors.

Example of a zoonosis?

Brucellosis - requires animal hosts. Brucellae are Gram-negative bacilli that have an animal reservoir and can cause chronic relapsing fever in humans Can be spread by direct contact - human to animal OR indirect contact e.g. eating infected unpasteurized milk and cheese

Which cancers are associated with the Epstein-Barr virus?

Burkitt lymphoma, (non) Hogdkin lymphoma, nasopharyngeal cancer

Histological sections are taken of the airways of a young man who died in a road traffic accident. He was a non-smoker and had no airway abnormalities. What cellular structures are most likely to be visible on microscopic examination of the respiratory bronchioles? A) Cartilage, smooth muscle and type II alveolar cells. B) Smooth muscle, goblet cells and ciliated epithelial cells. C) Smooth muscle, type I and type II alveolar cells. D) Type II alveolar cells, macrophages and ciliated epithelial cells. E) Macrophages, smooth muscle and serous glands.

C) Smooth muscle, type I and type II alveolar cells.

A 10-year- old boy is admitted with pneumonia. The pathogen is confirmed as Mycoplasma pneumoniae. He is commenced on intravenous clarithromycin, a derivative of erythromycin. Which is the mode of action of clarithromycin? A. Inhibition of bacterial cell wall synthesis B. Inhibition of bacterial efflux pumps C. Inhibition of bacterial protein synthesis D. Inhibition of nitric oxide synthase E. Stimulation of the immune system

C. Inhibition of bacterial protein synthesis (a Macrolide)

Which of the following statements is TRUE of C3b? A C3b is chemotactic for neutrophils B C3b is an anaphylatoxin C C3b opsonizes bacteria D C3b directly injures bacteria E C3b is the inactive form of C3

C3b opsonizes bacteria Bacteria coated with C3b adhere to C3b receptors on professional phagocytic cells. C5a, not C3b, is chemotactic for neutrophils. C3a and C5a, small peptides split off from the parent C3 and C5 molecules, are anaphylatoxins that trigger mast cell degranulation. There is no evidence that C3b directly injures bacteria. The inactive form of C3 is obtained by splitting the C3b to form iC3b, C3d, and C3dg.

Complement component C3 is cleaved by: A C3b B C3bBb C Factor B D Factor D E Factor H

C3bBb is the C3 CONVERTASE enzyme generated in the alternative complement pathway and is responsible for splitting off the small peptide C3a, leaving C3b as a residue. C3b is an opsonizing agent that assists phagocytosis and also contributes to some of the cleavage enzymes.

Which molecule best characterises the activation of interdigitating dendritic cells? A. CD8 B. CD25 C. CD28 D. CD40L E. CD80

CD40L

The genes coding for what region of the antibody have the highest rate of somatic mutation?

CDR - complementarity determining regions

How is GAS clinically manifested and what are the complications?

Can be divided into 1. local infections 2. invasive infections, and 3. POST-GAS disease 1. Local • Pharyngitis occurs 24-48 hours after exposure to GAS • Sudden onset of a sore throat, malaise, fever and headache. • The pharynx and tonsils may be erythematous with creamy/yellow exudates (fluid that filters from the circulatory system into lesions or areas of inflammation). 2. Invasive • Erysipelas: acute skin infection accompanied by lymphadenopathy, fever, chills and leukocytosis. Painful inflamed skin that is raised and clearly demarcated from the surrounding healthy skin. This can occur anywhere, but the legs are a frequent site of infection. • Cellulitis: similar to erysipelas but involves connective tissue as well as the skin. • Necrotising fasciitis can also occur - this is a deep infection of connective tissue that destroys muscle and fat. The bacteria enter through a break in the skin. Over a matter of hours there is swelling and the appearance of a spreading red or dusky blue skin discoloration, often with fluid-filled bullae. CLINDAMYCIN = treatment (for necrotising fasciitis) 3. POST a.) Rheumatic Fever (inflammatory disease of the connective tissue) may occur 3 weeks after pharyngitis --- autoimmune disease in which antibodies induced against streptococcal antigens (M protein) cross-react with human tissues b.) Glomerulonephritis - inflam. of kidney, edema, hypertension, proteinuria, hematuria ----also autoimmune *Complications? • Scarlet fever. This includes a 'strawberry' bright red tongue, fever and a characteristic rash (this is red, blanches upon pressure) that starts on the face and can spread to the chest, back and then the rest of the body.

Define "emerging infectious diseases" (doesn't necessarily have to be a "new" infectious agent)

Can be: • A recognised infection spreading to new areas or populations • Association of a known disease with an infectious agent • A "new" infectious agent • Change in an existing infectious agent • Re-emergence of an "old" infection through drug resistance or public health failure. Remember: To get established, each emerging agent must INFECTand TRANSMIT

What is moniliasis? Who is most at risk?

Candida infection. Moist area encourages Candida spp. Children and very old women (childbearing women have lactobacillus bacteria in vagina which keeps it acidic to prevent candia) Diabetics are more prone to the condition - candida is yeast, so it ferments sugar

1 limitation of HAART

Cannot eradicate latent HIV

Why do some viruses remain localised in the epithelium?

Cannot traverse the basement membrane in the mucosal layer

What is a tolerogenic environment?

Capable of producing immunological tolerance The gut and liver are tolerogenic environments during homeostasis.

Which test can you use to distinguish Streptococcus pyogenes from staphylococcus?

Catalase test

Legionella

Caused by Legionella pneumophila - gram-negative Can lead to: pneumonia-type illness called Legionnaires' disease and a mild flu-like illness called Pontiac fever

Which class of M proteins cause secondary complications, as well as the primary complications of "suppurative (pus) infections" and "glomerulonephritis"?

Class 1 M proteins can also go on to cause Rheumatic fever!

Closed (simple) fracture vs. compound fracture fracture site communicates with skin surface

Closed (simple) fracture: skin is intact Compound fracture: fracture site communicates with skin surface

How do dermatophytes cause infection?

Come into contact with a spore, & the spore will start to adhere to your nail, germinate & produce enzymes (keratinases) to break down the keratin into nutrients it can use a) Adhesion b) Nutrient acquisition c) Keratinase action

What does an "immune complex" often contain, besides antigen and antibody?

Complement proteins are often bound to the complex too

What is the lamina propria?

Connective tissue containing ACTIVATED immune cells

What are CD28 and CD80 molecules? How are they involved in T cell activation?

Costimulation - "2nd signal" molecules You need right T cell for the right peptide BUT that is not enough! You also need a host of other molecules on the cell surface which interact w / eachother , known as co-signalling molecules — all 2nd signal they are NOT antigen-specific they allow the response to occur once antigen specificity has been established What's the purpose of that? So immune system can REGULATE the response

Ig alpha and Ig beta molecules on either side of an antibody (BCR) are homologous to CD3 molecules on either side of a TCR. What do these molecules do?

Costimulation factors. They tell the cell that an antigen has bound to the receptor! Involved in cell-signalling.

S. aureus (virulent) & S. epidermis (harmless skin commensal flora) are indistinguishable under a microscope - so how can you differentiate between them?

Culture -> coagulase test S. aureus produces coagulase so it make a clot (coagulase turns fibrinogen into fibrin) Whereas s.epidermis does not.

In bacterial diagnosis, what is the advantage of CULTURE over microscopy? But what is a disadvantage of cultures?

Culture = more sensitive than microscopy + growing an organism allows you to do further testing e.g. antibiotic sensitivity testing Cons: slow & need to take culture before patient starts antibiotic treatment (or none will be left) + some bacteria can't be cultured on artificial media but only in eukaryotic cells like fibroblasts e.g. chlamydia

What's the difference between G-CSF (granulocyte colony-stimulating factor) and M-CSF (monocyte colony-stimulating factor)? What are G-CSF and M-CSF both?

Cytokines! G-CSF induces differentiation of stem cells into granulocytes. M-CSF induces differentiation of stem cells into monocytes.

cytopathic effect

Cytopathic effect or cytopathogenic effect (abbreviated CPE) refers to structural changes in host cells that are caused by viral invasion

Natural killer (NK) cells do NOT: A Respond to interferon B Contain perforin C Contain tumor necrosis factor (TNF) D Kill only by damaging the target cell outer membrane E Contain serine proteases

D Kill only by damaging the target cell outer membrane Perforin inserts into the target cell membrane thereby facilitating entry of TNF and granzymes into the target cell. Their activity is stimulated by interferon. They contain perforin, which is released on contact with a target, inserts itself into the target cell membrane, and creates a pore through which granzymes are transferred into the target cell. TNF produced by NK cells contributes to the killer function. NK cells contain a number of serine proteases of the granzyme family, one of which, granzyme B, can split procaspase 8 and thereby activate apoptosis in the target cell. Next question

A 28-year-old female sex worker is screened for possible HIV infection. It is found that she has antibodies which bind to the gp120 component of HIV-1. Which one of the following best describes the cell that secretes these antibodies? A) B cell. B) B lymphocyte. C) Plasma blast. D) Plasma cell. E) Immunocyte.

D) Plasma cell.

Which one of the following statements best applies to HIV? A) A novel human herpesvirus called HHV6 is thought to be critically involved in the aetiology of Kaposi's sarcoma. B) Infection of the infant may occur at around the time of birth but breastfeeding carries no risk of infection because breast milk does not contain the virus. C) The envelope of HIV contains exclusively virally encoded proteins. D) The gp120 envelop glycoprotein of HIV is involved in binding to CD4 molecules on susceptible cells. Additional co-receptors are also involved in HIV attachment and entry. E) The HIV particle is tough and thus withstands boiling and detergents.

D) The gp120 envelop glycoprotein of HIV is involved in binding to CD4 molecules

4. What are the specialized cell type involved in the entry of lymphocytes into lymph nodes called? A M-cells B Mesangial cells C Periarteriolar lymphoid sheaths (PALS) D High-walled endothelium of the postcapillary venules (HEV) cells E Selectins

D. The HEV cells in lymph nodes express vascular addressins which are recognized by homing receptors on lymphocytes passing through the afferent lymphatics and which mediate entry of the lymphocytes into the lymph nodes. Antigen is largely excluded from entering the body by the mucosal epithelial cells which have tight junctions and a protective mucus layer. However, specialized antigen-transporting M-cells interspersed between the gut columnar epithelium pass antigen from the gut to the underlying antigen-presenting cells. Mesangial cells are the phagocytic macrophages of the kidney. PALS is a tissue, not a cell. These PALS in the spleen consist predominantly of T-cells which surround the splenic arterioles. Selectins are a family of molecules, not cells. L-selectin is involved in entry of lymphocytes into lymph nodes and is the lymphocyte homing receptor which recognizes the HEV addressin molecule.

The part of an IgG antibody that binds complement component C1q is best described as the A. Complement receptor B. Heavy chain C. ɣC (gamma C) domain D. Fc E. Constant region

D. Fc

Viruses can be grown in tissue culture and sometimes produce a cytopathic effect (CPE). Which of the following is the single best method for demonstrating a cytopathic effect? A. Enzyme linked immunosorbent assay (ELISA) B. Immunoelectron microscopy C. Latex agglutination test D. Light microscopy E. Polymerase chain reaction

D. Light microscopy Cultured cells that are infected by most viruses undergo morphologic changes, which can be observed easily in unfixed, unstained cells by a light microscope.

Which is the single site in the body where the periarteriolar lymphoid sheaths (PALS) are present? A. Bone marrow B. Lymph nodes C. Mucosa-associated lymphoid tissues D. Spleen E. Thymus

D. Spleen PALS are a portion of the white pulp of the spleen, they surround central arteries in the spleen and are populated largely by T-cells.

What is the main replicative enzyme in DNA replication of viruses in a host cell? Which virus with a larger genome encodes its own DNA polymerase, rather than using the host's? Since ss retroviruses are RNA viruses whose genome alternates between RNA and DNA, what polymerase do they use? What then happens to the dsDNA in retroviruses? *What is the difference between RNA-dependent RNA polymerases and DNA-dependent DNA polymerases in terms of diversity of RNA vs. DNA viruses?

DNA-dependent DNA polymerase (transcribes viral DNA into DNA copy) Herpesvirus. ss Retroviruses: Reverse transcriptase transcribes the single-stranded viral RNA into double-stranded DNA. Reverse transcriptase is also known as "RNA-dependent DNA polymerase", because it copies ss RNA into ds DNA. The dsDNA then integrates into a host cell chromosome (now called a provirus). From this template, RNA genomes can now be transcribed to produce progeny virus e.g. HIV So you have reverse transcription acting as soon as the virus has uncoated!! Then integration of dsDNA into host cell chromosome -> then transcription & translation *RNA-dependent RNA polymerases have no proof reading mechanisms so any errors become immortalised ---> "quasi-species". DNA-dep DNA polymerases have a proof-reading mechanism so misincorporation rate is much lower so less genetic diversity

What are neutrophil defensins?

Defensins are peptide ANTIBIOTICS present in the granules in extremely high concentration

List 2 deficiencies in neutrophil function, that are examples of primary immunodeficiencies:

Deficiencies in neutrophil function: 1. leukocyte adhesion deficiency (due to a defect in the 2 integrin gene) 2. failure of intracellular bacterial killing (chronic granulomatous disease due to a mutation in any one of the genes encoding the components for the NADPH oxidase, including several autosomal genes but most commonly the gp91phox gene on the X chromosome)

How does HIV cause disease?

Depletion of CD4+ T cells by direct cytopathic effects & apoptosis after infection. - CD8+ T cell killing of those infected activation and apoptosis of uninfected cells - Loss of CD4 T helper cell function impairs the adaptive immune response & leads to generalised immune suppression. • CD4 and CD8 cell counts are the individual measures of the immune systems used for monitoring patients.

Why are some fungi, which are organisms of low virulence, now causing severe illness in some people?

Due to advances in the treatment of cancer, patients are becoming more severely immunocompromised and organisms of low virulence, for example some fungi, are now causing severe illness in these patients.

What type of bond links both the alpha and beta chains of TCR heterodimers? This is also the type of bond between the 2 identical heavy chains on an antibody, and between the 2 identical light chains on an antibody!

Disulphide

Graft versus host (GVH) disease 1. Is a characteristic feature of liver transplantation 2. Can only occur if donor & host are mismatched at the HLA locus 3. Occurs when donorderived B cells produce antibodies 4. Often targets the host gastrointestinal tissue 5. Can be prevented by deleting donor T cells from the allograft

Donor T cells attack host tissues. When the graft contains HLA-incompatible T cells (e.g. a bone marrow or peripheral blood haematopoietic stem cell graft) these can attack host tissues, especially skin, lung and gut, leading to life-threatening GVH disease. The risk of this occurring can be reduced by removing the T lymphocytes prior to transplantation and using immunosuppressive drugs. Need to balance immuno-suppression (to prevent graft vs host) but also keep immune system alive so it can protect host against disease. 4. Often targets the host gastrointestinal tissue

Diff between antigenic shift & drift?

Drift = gradual accumulation of mutations Shift = much more drastic : Reassortment of genome segments upon co-infection (influenza)

A 35-year-old man visits his GP because he has weight loss, constantly feels hot, and has a fine tremor of the hands. Upon examination he is found to have a diffuse goitre. Which one of the following antibodies is most responsible for the signs and symptoms in this patient? A) Anti-thyroglobulin. B) Anti-thyroid peroxidase. C) Blocking antibodies to TSH receptor. D) IgG. E) Stimulatory antibodies to TSH receptor.

E) Stimulatory antibodies to TSH receptor.

Infectious mononucleosis

EB virus infects B Cells to cause infectious mononucleosis. Infected B cells look immortalised in tissue culture...later on the cytotoxic T cells attack the person's B cells!

The 3 es of cancer immunoediting

Elimination, equilibrium, escape 1. Elimination: immune cells recognise antigens on surface of cancer cells, recruiting other cells of the immune system to kill the nascent tumour 2. Equilibrium: (between tumour proliferation & apoptosis) Some tumor cells may be resistant enough to withstand the attack by immune cells and enter into a stage of dormancy 3. Escape: when the tumour cells win the battle & a clinically detectable tumour forms

Which 3 cells secrete inflammatory mediators?

Eosinophils, basophils and mast cells

What is the difference between endogenous and exogenous infections?

Endogenous: Infections caused by the normal flora of the body penetrating to sites that are sterile e.g. peritoneal cavity, or by overgrowth of the normal flora due to increased host susceptibility. Exogenous: Infections caused by microbes gaining entry to the body from OUTSIDE

What are the 2 methods of penetration/virus entry for enveloped viruses, and the 1 method for non-enveloped viruses?

Enveloped: 1. Fusion 2. Endocytosis AND fusion (@ low PH) Non-enveloped: Endocytosis

A 22-year-old army recruit who lives in local military barracks is admitted to hospital with a fever and a large boil over his left upper thigh. He had been prescribed an antibiotic tablet called 'flucloxacillin' by his GP for 7 days that had made no difference to the infection. He has a temperature of 38.2oC, a heart rate of 92 bpm and low blood pressure. He has no past medical history, but he reports that 2 other recruits in his dormitory had similar problems with abscesses after an assault course exercise a week earlier. Why do you think the patient may not have improved with the flucloxacillin?

Even though bacteria sensitive to flucloxacillin, may still have failed due to poor penetration of antibiotics into an abscess.

MALT, GALT, BALT?

Every organ has its own specialised populations of immune cells Mucosa-associated lymphoid tissue (MALT) BUT MALT) is further specialised: • Gut-associated lymphoid tissue: GALT • Bronchus-associated lymphoid tissue: BALT etc

ADVANTAGES of the normal flora?

Exclusion of potential pathogens (colonisation resistance), facilitating maturation of the host immune system, metabolisms of bile, providing nutrients to host

Dimorphic funghi - 3 examples in USA that pick up during tornado/storm + are breathed in to infect lungs & cause flu-like symptoms

Exist as mould in enviro but then can switch to yeast when temp goes above 36 c --- usually more severe symptoms in immunocompromised patients 1. Coccidiodes immitis: produces SPHERULES to protect the yeast as it's multiplying inside the lungs! 2. Histoplasma capsulatum: 3. Blastomycosis: 2+3 these yeast become intracellular to protect themselves (go into macrophages & neutrophils)

Mucoraceous mould infection

Extremely rapidly growing mould (fill whole agar plate in 24 hours) - medical emergency Huge spores grow up into sinus into eye socket & into brain! - can get this infection on wounds too -common in patients on long-term use of steroids because these suppress the immune system - in diabetes common too - lots of iron in bloodstream due to ketoacidosis - the mould loves iron

What do follicular dendritic cells do? FDC

FDCs present antigen (in the form of an immune complex) to B cells. B cells with high affinity receptors will recognise these antigens in the germinal centre and get selected for and go on to differentiate into plasma or memory B cells. B cells w low affinity will die by apoptosis in germinal centres. Phagocytic macrophages will remove them.

What does Fc stand for and what does this part of the antibody contain? What happens here?

Fc = fragment crystallisable. Contains the biological activity effector part of the antibody. Fc is where the antibody binds & activates complement! It also attaches to Fc receptors on other cells here.

What are ectoparasites?

Fleas & lice -- live on the skin but not within the body

Where is Strep. pyogenes found in the body?

Found all over the body but enters via the oropharynx

Key site of B cell differentiation (including clonal selection, somatic hypermutation, selection & class switching)? *Process AKA "Affinity maturation" Why is it limited to this area?

Germinal centre in lymph node. V dangerous process so it can be isolated & tightly regulated-so only the best B cells (the ones w the highest affinity for the antibody) are allowed to differentiate into plasma cells or memory B cells

Which 2 antibiotics act on the cell wall, & how do they differ? Which 1 is capable of treating MRSA?

Glycopeptides & B-lactams 1. B-lactams inhibit enzymes which synthesise peptidoglycan --> cell lysis 2. Glycopeptides PHYSICALLY obstruct the cell wall from coming together (not acting on enzymes) Only glycopeptides will treat MRSA! Because MRSA enzymes which synthesise peptidoglycan have mutated

Which Baltimore group of viruses does HIV belong to?

Group 6 (retroviruses) -- although 7 is also retroviruses!

What are retroviruses? Which Baltimore group?

Group 6! Retroviruses: RNA viruses whose genome alternates between RNA and DNA. Viral ssRNA is transcribed into dsDNA by reverse transcriptase. The dsDNA then INTEGRATES into a host cell chromosome (now called a provirus). RNA genes can then be transcribed to produce the proteins & enzymes for a progeny virus! (transcription, translation, core particle assembly, budding)

How is Streptococcus classified?

Group A streptococcus (GAS) Streptococci can be classified according to their haemolytic properties. BETA HAEMOLYSIS - this is complete haemolysis of RBCs in the agar medium, resulting in a clear halo around the colonies. Beta haemolytic streptococci can be further sub-divided based on their carbohydrate antigens (groups A-T; S. pyogenes is group A). *Alpha haemolysis is partial haemolysis. It causes iron reduction in RBCs, giving a green halo around the colonies (it is green as the bacteria converts Hb to methemoglobin).

What are embryonated chicken eggs important for?

Growing viruses used for vaccines e.g. MMR, influenza (much faster method than tissue culture)

Name 3 bacteria with polysaccharide antigens ! - these require what special type of vaccine?

H. influenza N. meningitidis S. pneumoniae Conjugate vaccines! Where the polysaccharide antigen is combined w a protein carrier so they can be more easily recognised

How HBV develops into HCC (cancer)

HBV infection causes chronic hepatitis -- inflammation carries on 20-25 yrs cirrhosis 25-30 yrs HCC - hepatocellular carcinoma

2 successful drugs for Hep C Most common group infected?

HCV protease & polymerase inhibitors - now a curable infection (but expensive) IV drug users

Eisophils secrete a basic protein which damages....

HELMINTH membranes

Which virus binds to TAP?

HSV - Herpes Simplex! ICP47 binds to TAP and prevents peptide translocation into the endoplasmic reticulum.

Which 2 diff shapes can viral capsids be?

Helical nucleocapsid (with helical symmetry) AND Spherical capsid (with icosahedral symmetry)

Immunological cells enter the peyer's patches of the gut from blood via HEV (high endothelial venlues) , but what does the liver have instead of HEV?

Hepatic sinusoids

Which virus apart from HBV can infect liver & cause hepatocellular carcinoma? 1. Epstein-Barrvirus 2. Hepatitis A virus 3. Hepatitis C virus 4. Hepatitis E virus 5. Yellow fever virus

Hepatitis C virus * Epstein-Barrvirus -- usually causes mono, inflammed throat, can cause hepatisis but not cancer all others can cause hepatitis but not cancer

Old people have an increase in visceral abdominal adiposity --> effect on immunosenescence?

Higher numbers of immune cells (macrophages and T cells) are found in adipose tissue and this correlates with increased inflammation.

What is the streptococcus pyogenes capsid made from?

Hyaluronic acid **note only some GAS produce a capsule

Which cytokine is required for B lymphocyte differentiation to begin? A. IL-2 B. IL-4 C. IL-7 D. IL-13 E. IL-22

IL-7

Which are the only Ig classes which are NOT monomers? IgM antibodies are more efficient at agglutination than IgG antibodies. T/F?

IgG = pentamer IgA = dimer TRUE

Blood group antibodies are mainly of what class? What do you know about this molecule's affinity & avidity? Which 2 Ig classes activate complement?

IgM Although IgM Fab has a low affinity for antigen, its pentameric structure (and thus 10 arms) give it a relatively high avidity! IgM and IgG activate complement. IgG can also opsonize bacteria for phagocytosis.

Antibody detection is limited in that it won't give you an idea of when the infection was acquired, but the presence of one Ig does suggest that it is a more acute infection - which Ig is this? Antigen testing can pick up all groups of N. meningitis except?

IgM = more acute infection Except group B

Which 2 antibody classes are usually expressed on the surface of B cells?

IgM and IgD

Difference between immunosenescence & cellular senescence

Immune Senescence: the process of deterioration of the immune system with age Cellular or replicative senescence: the loss of a cell's ability to divide and grow

Beta Lactam antibiotics No activity against which bacteria?

INHIBITION OF enzymes involved in CELL WALL SYNTHESIS - shared structural feature, the beta-lactam ring. • 3 groups - Penicillins - Cephalosporins - Carbapenems - No activity against atypical organisms • Mycoplasma • Legionella • Chlamydia

Key fungal infections to remember in the immuno compromised (2 subgroups) vs. the immunocompetent (3 subgroups):

Immunocompetent: •1. Superficial - Candida / Dermatophytes •2. Dimorphic - Coccidioides / Histoplasma / Blastomyces • Immunocompromised: • 1. Invasive - Aspergillus / Candida •2. Mucoraceous mould infection • 3. AIDS defining - PCP (P. jirovecii infection) / Cryptococcus

When does adaptive immunity peak in the life cycle? vs. how does innate immunity change over time?

In contrast to innate immunity, adaptive immunity is not completely mature at birth, but peaks at puberty and declines thereafter. Innate immunity declines steadily from birth.

Changes to T cell compartment with age

Increasing numbers of: Memory T cells Falling: Naïve T cells Co-stimulatory molecules TCR signaling Cytotoxicity Proliferation IL-2 production Telomere length

How could increased homeostatic proliferation in old people be linked to the increased presence of autoimmune diseases?

Increased homeostatic proliferation could favor the selection of autoreactive T cells and the differentiation into memory-like cells and thereby provide a foundation for overcoming peripheral tolerance mechanisms and inducing an autoimmune disease.

What do we use these 2 stains for? -India ink -Silver stain What shape organisms are not visible by gram stain? So what stain should you use to diagnose syphilis?

India ink: cryptococcus Silver stain: pneumocytus jiroveci *cryptococcus is a fungus that is the most common cause of meningitis in AIDS *pneumocytus jiroveci is also a fungus that causes pneumonia in the immunosuppressed Spiral - cannot see on Gram stain --> syphilis - spirochetes can only be seen with immunoflourescent stains!

Microscopy isn't just used to look @ pathogens -- also for cell counts! White blood cell count in CSF in an infection? Neutrophil count is elevated in what 2 conditions?

Infected: WBC >6/mm3 = evidence for infection High neutrophil count: >250/mm3 in 1.ascites (fluid accumulates in abdomen in chronic liver disease, and the fluid can get infected) 2. pleural/joint fluid (also cause for concern)

IDC vs FDC

Interdigitating Dendritic cells AKA Langerhans cells : present antigens to T Cells - Take up antigens in tissues (APCs) and migrate via afferent (veiled cells) to lymphoid organs where they interact with cells adaptive immune system - express MHC class II FDC: present immune complexes to B cells

Name 3 groups of antibiotics which interfere w protein synthesis, and 3 groups of Beta-lactams which interfere w cell wall synthesis? Which group interferes w nucleic acid synthesis? Which bacteria is this one most effective against?

Interfere with protein synthesis (70s ribosome) 1. Tetracyclines 2. Macrolides (erythromycin) 3. Aminoglycosides (gentamicin) Beta-lactams : 3 groups - Penicillins - Cephalosporins - Carbapenems Intereferes w DNA/RNA synthesis: Quinolones (Ciprofloxacin) (inhibit enzymes DNA gyrase & toposiomerase) The greatest activity of quinolones is against gram-negative bacilli (such as E.coli).

Which 2 protozoa disrupt the host's immune response by invading macrophages? Which 2 protozoa disrupt metabolism? Which protozoa induces granuloma formation like TB?

Invade macrophages: toxoplasma & leishmania Disrupt metabolism: Giardia cysts deconjugate bile salts -> can't digest fats Trypansome cruzi -> cardiac failure Shistosomiasis induces granuloma formation

KSHV

KSHV proteins inactivate (Tumour Suppressor Gene) stage in cell cycle. So you get indefinite cell replication. If the person is not immunocompromised, they might just get a skin disease. Immunosuppressed (e.g. HIV patients) will get it invading internal organs!

Why does M.TB have mycolic acid in its cell wall? What's good about being wrapped in fat?

Keeps all the moisture in so it won't get dehydrated. Mycolic acid = long chain hydrophobic fatty acids. Acid-fast stain targets mycolic acid - only stains M.TB and not other bacteria - so can identify it. Penicillin doesn't work on M.TB, because it can't get in through the mycolic acid! ---> So most antibiotics for TB target the mycolic acid

Organ with longest graft survival rate?

Kidney (kidney along w pancreas transplant is even better) Skin is most difficult. Autograft for skin is most common. + Skin completely regenerates. Skin is highly vascularised so v easy for immune cells to get to it.

Phagocytes are mainly made up of what 2 cell types?

Macrophages and neutrophils

What does TLR4 detect?

LPS in gram-negative bacteria e.g. gram negative rod E-Coli or gram negative cocci Neisseria meningitidis

List 3 effects of LPS on the host defence mechanisms that can lead to septic shock What is septic shock?

LPS: 1. activates macrophages to release cytokines such as TNF 2. activates B cells 3. activates the complement, kallikrein & coagulation cascades Septic shock: when blood pressure drops to a dangerously low level after an infection. - a complication of sepsis (symptoms of septic shock may develop if sepsis is left untreated) Sepsis = organ injury or damage due to infection respiratory rate >22 breaths per minute, systolic blood pressure <100mmHg, altered mental status. *N.B. Most cases of septic shock are actually caused by gram-positive bacteria,[8] followed by endotoxin-producing gram-negative bacteria **e.g. superantigens in gram pos c.f. strep pyogenes

Leishmania

Leishmania is a diphasic parasite that completes its life cycle in two hosts: a sand fly that harbors the flagellated extracellular promastigote form and a mammal in which the intracellular amastigote parasite form develops. Often in dogs in southern Europe, Africa, Asia, South and Central America

1 side effect of antiretroviral therapy

Lipodystrophy: abnormal fat distribution

Signs of acute hepatitis B infection?

Liver damage results in failure to excrete pigments derived from haemoglobin, resulting in dark urine, pale stools and clinical jaundice.

What is 1 of the major changes to the T cell surface as we age?

Loss of co-signalling molecules CD27 and CD28

Invasive Aspergillosis Opportunistic pathogens -immunocompromised 1. How to diagnose? 2. Symptoms?

Lungs become infected & v diff to treat & diagnose *sputum sample : v small chance that u will find the mould in it! *but can use at ELISA test to look for polysaccharide present in fungal cell wall *can use CT scan also to look for "Halo sign" = growing mycelial network in lung around a solid lesion *flourescent stain to see hyphae w microscope Symptoms •neutropenic •fever non- responsive antibiotics >4 days •cough, chest pain or discomfort or breathlessness • haemoptysis

As well as activated T cells (CD4 + CD8), what other cells is the lamina propria rich in?

MAST cells. These are important in our response against PARASITIC WORM INFECTIONS. C.f. Hypersensitivity type 1: cross linking of IgE ab on the surface of mucosal mast cells by worm antigens leads to mast cell degranulation --> release of histamines --> powerful inflammatory reaction

Largest contribution to genetic susceptibility is the .... genes Many autoimmune diseases more common in men than in women. T/F?

MHC genes e.g. Risk of developing ankylosing spondylitis is 90 times higher if you have HLA-B27 False. Many autoimmune diseases more common in women than men

The severity of graft rejection is partly determined by the number of genetic differences in the ...... molecules between the donor and recipient Most important alleles for matching?

MHC! MHC molecules are the most POLYMORPHIC genes in the human genome (differ a lot from 1 person to another). The polymorphism is specific to the peptide binding site of the MHC. Therefore, the best way to prevent graft rejection is to try to reduce these differences and select donors and recipients as closely matched for the different MHC alleles as possible. The most important alleles for matching are HLA-DR, followed by HLA-B and then HLA-A.

MacConkey agar *What is a tellurite plate used to culture?

MacConkey agar is a selective and differential culture medium for bacteria designed to selectively isolate Gram-negative and enteric (normally found in the intestinal tract) bacilli and differentiate them based on lactose fermentation.[1] The crystal violet and bile salts inhibit the growth of gram-positive organisms which allows for the selection and isolation of gram-negative bacteria. Enteric bacteria that have the ability to ferment lactose can be detected using the carbohydrate lactose, and the pH indicator neutral red - helps you distinguish between those bacteria which ferment lactose (e.coli) & those which cannot (salmonella, shigella) *Tellurite plate: used to culture Corynebacterium diphtheriae - the cause of diptheria. FYI: C.diphtheriae is found in the oropharynx & has an exotoxin which inhibits protein synthesis in heart & nerve cells.

What are Kupfer cells?

Macrophages (just have a special name cos they're macrophages in the liver). They remove unwanted products that flow in blood through the HEPATIC PORTAL VEIN from the intestines. If liver cells encounter an infection/antigens, many lymphocytes will infiltrate (fast recruitment like with skin immunity)! But this can be problematic (e.g. viral hepatisis) when too many immune cells take over & there are not enough liver cells for the liver to keep functioning --> liver failure

What is the consequence of the accumulation of oligoclonally (derived from only a few clones) expanded, functionally impaired T cells in old people? What else increases homeostatic proliferation?

Many of these clones are specific for antigens that are continuously present, such as Cytomegalovirus (CMV) and Epstein Barr Virus (EBV). This accumulation of suboptimal highly differentiated CMV-specific T cells causes overcrowding of the memory T cell pool, leading to the constriction and loss of infrequent memory T cell populations (eg influenza and VZV). These age associated changes contribute to the inability of the aged immune system to respond to new antigenic challenge and mount poor responds following vaccination. Homeostatic proliferation is greatly increased in lymphopaenic conditions, such as those induced by irradiation, chemotherapy or HIV infection.

Why is SCID generally not detected before the age of approximately 3 months?

Maternal IgG passed through placenta. Present until approximately 3-5 months

Critical community size

Measles vs. chickenpox For an infection to persist in a population, it is necessary for at least one infected individual to pass that infection onward — and if all the infected individuals in a population fail to transmit, then the infection will go locally extinct, or "fade out" in that population. Formally, we can define the Critical Community Size as the smallest population size for which any given infection does not go extinct. Measles: One of the most classic exhibitions of the Critical Community Size has been the observation that measles infection, in the era preceding the development of the measles vaccine, was always present in cities of greater than 3-500 thousand, but was frequently absent in smaller towns — and more frequently absent the smaller the town.

3 examples of bacteria which have developed efflux pumps?

Mef (macrolides), Tet A-E (tetracyclines), - MexAB-OprM(carbapenems)

These complement proteins form the .......... C5b,6,7,8,9

Membrane attack complex (MAC)

Sebacious glands secrete sebum....why do we find microbes here on the skin?

Microbes use sebum as a nutrient

Cryptococcosis - Opportunistic pathogens -immunocompromised

Migrates to brain & causes meningitis CNS disease: •Meningitis (85%), meningoencephalitis, cryptococcoma •HIV CD4 count <200 cells/mm3 Pulmonary cryptococcosis: •May experience self limited pneumonia •Invasive chronic pulmonary disease may occur - see lesions in brain - then take lumbar puncture CSF (invasive procedure) - look for cryptococcal capsid (antigen-testing)

2. What makes Staphylococcus aureus a serious pathogen? Can you name some of its virulence factors?

Mnemonic THE STAPH CELL Toxic-shock staphylococcal toxin (TSST) Haemolysin Enterotoxin Staphylokinase Teichoic acid (A) protein A Penicillinase Catalase/coagulase Exfoliatin Leukocidin Lipase

Ziehl-Neelsen stain is used for what bacteria?

Mycobacterium, because it's cell wall is too THICK and LIPID-RICH for gram staining to work

What is a very fast way to view viruses in a microscope?

NEGATIVE STAIN using an electron microscope - allow you to see SURFACES (a thin section will show you the inner structure)

What are the 3 most important functions of antibody?

NOC 1. Neutralisation 2. Opsonisation 3. activate the Complement 1. Neutralisation: directly inhibit bacterial toxins or viruses from binding to their target cell 2. Opsonisation: stimulate phagocytosis and killing of bacteria & protozoan parasites by macrophages & neutrophils 3. Activate the complement via the classical pathway --> inflammation, opsonisation, and MAC complex cell lysis

Naive B cells express which two Ig classes as membrane receptors? IgM is monomeric when used as a BCR, but how is it found when plasma cells secrete it as a soluble molecule? Why does this mean it now has a very high avidity? IgA is in one form in circulation but another in mucosa. Which forms?

Naive: IgM and IgD IgM monomeric ---> pentameric (this means it has 10 antigen-binding arms! = high avidity!) *N.B. IgD is never secreted as a soluble molecule - can never be found in circulation (only on BCR) IgA in circulation: monomer IgA in mucosa: dimer

What technology do you need if you want to sequence genomes?

Next Generations Sequencing: Solexa or Illumina method Principle: Use a dideoxy (not deoxy) because then one of the hydroxy (OH) groups is blocked -- uses chain termination in the same way NNRT antiviral drugs do!

Can latent viruses be eradicated?

No

Although there is a fall in the number of B cells, the no of what (x) doesn't seem to change? But how does the quality of (x) change? (6)

No of antibodies stays the same. But the quality of the antibody response changes! isotype change e.g. in the young, antibody responses to influenza vaccine = primarily of the IgG1 subclass, whereas in the elderly they tend to be of the IgG3 subclass. IgM memory reduces w age (responsible for protection against common bacterial polysaccharide T-independent responses) ----> explainst he increase in pneumococcal infections in the elderly Somatic hypermutation is reduced B cell diversity is reduced ----> decrease in antibody affinities and Ig repertoire increase in autoantibody production especially rheumatoid factor, ant-dsDNA, antihistones and anticardiolipin antibodies. A diminished ability to produce antibodies to vaccines (e.g. influenza).

Do all organs have the same populations of immune cells?

No, every organ has its own specialised population of immune cells reflective of its microbiome e.g. gut has it's own lymphoid tissue (GALT) cos it has such high exposure to microbes! -- IgA production -- intra-epithelial lymphocytes -- Peyer's patches (localised lymphoid tissue)

Describe the physical forces between lymphocyte receptors and their antigens

Non-covalent, and therefore REVERSIBLE *although in practice, antibodies stay bound to an antigen for life anyway

How is the "release" stage of the viral replication cycle different for enveloped viruses vs. non-enveloped viruses?

Non-enveloped viruses: need the dying cell to disintegrate so that they can get released! Enveloped viruses: nucleocapsids attach to viral proteins inserted in the cell membrane --> & the new virion 'buds' from the still viable cell

Why is scarlet fever rare in people with a strep pyogenes infection? *Which other common toxin is encoded in the same way?

Not all virulence factors are encoded by the bacterial chromosome; some exotoxins are PHAGE ENCODED. (The phage then infects the bacteria, live within it, and allow that virulence factor to be expressed). Only some strains of strep pyogenes have bacteria which contain the phage that encodes for the erythrogenic toxin! (only the erythrogenic toxin causes scarlet fever) *Cholera toxin is also phage encoded!

Induction & effector phase

Note that adaptive immune responses do not generally START outside lymphoid tissue. Antigen & APCs go from tissue to lymph node ( to facil. B+T cell differentiation process etc) but then go back to that infected tissue! 1st induction, then effector phase when they go back to the tissue The antigen in the lymph nodes stimulates an adaptive immune response (the inductive phase) , and the responding lymphocytes then migrate back to the tissues (effector phase).

R0zero: basic reproductive number

Number of cases arising from a single case e.g. Measles 12 - 18 Polio 6 Pertussis 12 - 18 Smallpox 5 - 7 Ebola 1.5 - 2.5

Diff between getting infected with Hep B virus by horizontal vs. vertical transmission?

Only 6% of children infected after age 5 become chronically infected, whereas 90% of infected infants (from mother) become chronically infected

In which cell types does regeneration occur? And only IF.....?

Only in tissue with labile & stable cells. & only if the supporting architectural framework ie basement membrane is still intact! (therefore pure regeneration after injury is v rare)

What are some common infections and cancers seen in people with AIDS?

Opportunistic infections: Viruses eg cytomegalovirus retinitis Bacteria eg tuberculosis Fungi eg Pneumocystis jiroveci, cryptococcus Parasites eg cerebral toxoplasmosis Neoplasia: Cervical cancer - papillomaviruses B cell lymphomas - Epstein-Barr virus Kaposi's sarcoma - HHV-8

What do streptococci in your mouth produce and how can this protect against a certain type of bacteria? Tell me more about that type of bacteria? How does cilial action count as a "host defense" in the respiratory tract?

Oral streptococci produce H2O2 (hydrogen peroxide), which can kill COLIFORMS! Coliforms are rod-shaped gram NEG e.g. E.Coli, which can ferment lactose Cilial action = mechanical removal of trapped matter in the respiratory tract

New approach to overcoming donor shortage and organ rejection?

Organ cloning using differentiated stem cells -- espesh advancements using acellular organ scaffolds as a substrate for differentiating stem cells

Difference between osteoblasts & osteoclasts?

Osteoblasts lay down bone, whereas osteoclasts break it down clast: multinucleate

How come the complement system is not normally triggered by our own cells?

Our own cells have complement inhibitory molecules on their surfaces

The pathogenesis of severe malaria is a consequence of A. Reduced circulating levels of inflammatory mediators B. Macrophage activation C. Overproduction of TNF, IL-1, IL-2 and IFNɣ D. Toxins produced during infection E. Activation of the host's immune system

Overproduction of TNF, IL-1, IL-2 and IFNɣ

Microbial diagnosis: Urine dipstick screening has an excellent NPV(99%) but a low .... (50%) NPV isn't as good in which 3 groups though? What 2 things does urine dipstick screening look for?

PPV is only 50%. It allows rapid exclusion of infection, at least. NPV is lower for pregnant women, children and people with abnormal urinary tracts. Looks for a.) nitrites (many bacteria convert nitrate into nitrite) b.) leucocyte esterase

Enterococci look like a combo of strep and staph - explain

Pairs & short chains

Helminths

Parasitic worms : large multicellular organisms! Most reside in the intestines (but exceptions e.g. schistosomes in blood vessels) All helminths produce EGGS for reproduction -- resistant to environmental conditions!

How is GAS spread?

Person to person via respiratory droplets - by hands or fomites. "Auto-inoculation" can also occur if someone touches other parts of their own body - spreading it around their own body

Why is the presence of catalase & glutathione peroxidase particularly important people who are suffering from an infection? Which enzyme (also protective against ROS) do neutrophils have that macrophages don't?

Phagocytosis requires OXYGEN - it is an aerobic process. The intracellular killing (in the phagolysosome) involves reducing oxygen to produce reactive oxygen species (ROS). Catalase & glutatathione peroxidase help to remove these ROS! (Excess ROS can damage host tissues e.g. blood vessels in arteriosclerosis) Neutrophils also have myeloperoxidase.

In systemic virus infections, the disease is distant from the portal of entry. Give 2 examples

Polio: enters via gut - disease in central nervous system Measles virus: enters via respiratory tract - disease is generalised

Which 2 antibiotics work by injuring the plasma membrane?

Polymyxin B (antibacterial) or fluconazole (antifungal). - injure the plasma membrane thus changing permeability & causing cell lysis due to loss of metabolites *can be toxic tho as humans also have outer cell membrane!

Why would you immuno-suppress a host before a transplant?

Prepare the host (by chemo, radiation) so the immune system is so suppressed that it will not attack the graft.

Primary viraemia: Secondary viremia:

Primary viraemia: viral replication at the original site of entry --> return to blood Secondary viremia: return to blood stream after virus has replicated in other tissues, in organs distal to the site of entry

Difference between productive & latent infections?

Productive infection: virus replication with the production of virions. Latent infection: a non-productive infection - viral genome present but no obvious effect on cell function. No virus replication.

What do the RAG (recombination activating genes) genes code for?

Recombinase Enzymes that RECOGNISE a stretch of nucelotides called an RSS (recombination signal sequence) where the V, D and J gene segments are.....these enzymes run the whole antibody recombination process

Define 'regeneration' & compare with 'repair'

Regeneration: The replacement of injured cells by PARENCHYMAL cells of the same type --> damaged area of tissue becomes 'normal' Repair: The replacement of injured cells by FIBROUS CONNECTIVE TISSUE --> damaged area of tissue becomes a 'scar'

What is Tinea capitis?

Ringworm of the head - caused by a dermatophyte fungus (dermatophyte means skin funghi) - Athelete's foot = a related infection

Which part of the innate immune system changes as we age and which stays the same?

Same: PRRs -- Many innate pattern recognition receptors (PRR) for bacterial components are NOT AFFECTED by ageing Change: Declining neutrophils: * Aged macrophages and neutrophils have impaired respiratory burst & reactive nitrogen intermediates as a result of altered intracellular signalling, rendering them less able to destroy bacteria. *Aged neutrophils are also less able to respond to rescue from apoptosis. Natural killer (NK) cells: *NK cells = less cytotoxic & make less IFNγ. *but more of them! + produce more IL-4 Complement: less comp proteins! The innate immune system also contributes to alterations in acquired immunity with ageing...... Dendritic cells (DC) & macrophages = less able to stimulate T and B cells.

What is the SASP?

Senescent cells of the immune system, adipose tissue, endothelium and stromal cells (fibroblast) develop a senescent associated secretory phenotype called the SASP. The SASP consist of 40-80 factors involved in tissue remodelling, inflammation, chemotaxis and cell migration and cell signalling. The SASP contributes to ageing by disrupting tissue structures and inducing inflammation. SASP factors are not only secreted by senescent cells but they also act in an auto- and paracrine fashion to maintain and induce senescence in cells and as such can drive senescence and immunosenescence.

Describe 2 different sources of epidemiological data

Seroepidemiology - e.g. testing serum to find prevalence of HIV or hepatitis B in a population. Molecular epidemiology - genotyping measles virus to determine the origin of the virus i.e. is it a strain endemic to the UK or has it been introduced from another country.

Why is the antibody response said to be polyclonal? (not monoclonal)

Since microbes have many different antigens, each with several epitopes, the antibody response is said to be polyclonal. Antibody responses generated in vivo, even against a single antigen, are polyclonal : they are complex and variable mixtures made up of many different antibodies each with a slightly different sequence and antigen affinity. However, it is possible to make monoclonal antibodies, which consist of a single unique protein. These molecules have very precise specificity for their targets, and have found widespread use as tools for cellular biology, as diagnostic tools , and most recently as "biological drugs".

Viruses localised to which area have a longer incubation period & why? a. respiratory + intestinal b. skin

Skin: longer incubation period Entry, infection & disease all happen @ same site

What is a somatic mutation?

Somatic mutations differ from germ line mutations, which are inherited genetic alterations that occur in the germ cells (i.e., sperm and eggs). Somatic mutations are frequently caused by environmental factors, such as exposure to ultraviolet radiation or to certain chemicals.

What are bacterial endospores?

Some bacteria e.g. Bacillus & Clostridium have endospores within the cell which protect their DNA --> make them v resistant to environmental conditions! ---> explains why c.difficile is so persistent in hospitals & care homes

How do some of the most successful monoclonal antibody cancer treatments work?

Some of the most successful monoclonal antibodies can activate cancer-specific immune cells in vivo. This occurs by antibody binding to negative regulators, such as PD1 and CTLA4, that are expressed by cancer-specific immune cells. The blocking of negative regulation enables the immune cells to get activated and attack cancer cells.

Peyer's patches

Specialized lymph nodes right underneath the wall of the gut Where T cells drive B cells to become IgA producing (class switching) -like normal lymph nodes, have specific areas for B and T cells - but also have M cells providing a source of antigen entry

Gram positive cocci arranged in clusters

Staphyloccocus (Gram positive cocci in long chains is streptoccus pyogenes)

Sore throat is mostly caused by viruses, but what is the principal bacterial causative agent of pharyngitis?

Streptococcus pyogenes

Hep B vaccine? - what type of vaccine? - what antigen does it contain? - how is it administered?

Subunit vaccine containing HBsAg as the immunising antigen - Intramuscular injection, three doses to be given at 0, 1 and 6 months - Protection is proportional to the amount of anti-HBs produced

Treatment of hepatitis B infection? Which antiviral drugs are used?

Supportive management of acute HBV, but antiviral treatment for chronic HBV - Goal is to suppress HBV replication, prevent the progression of liver disease Interferon alpha (α-INF) & HBV reverse transcriptase inhibitors (lamivudine, telbuvidine, entecavir and tenofovir) Fulminant hepatitis may require liver transplant

What will a culture of cells inoculated with RSV show?

Synctia! In the inoculated culture large syncytia of multinucleated giant cells (clusters of fused cells) are visible. This is the characteristic cytopathic effect caused by the replication of syncytium forming viruses such as RSV and measles.

Multiple sclerosis & Type 1 diabetes are ..... cell driven attacks

T cell driven attacks! MS: T cell driven attack of the central nervous system (CD4 T cells infiltrate CNS, activate macrophages, B cells...etc) --> Demyelination in brain and spinal cord --> Slower nerve conduction, axonal injury, neurological dysfunction Autoantibody = myelin basic protein Diabetes type I: T cell driven attack of the insulin-producing pancreatic cells

Discuss the pathophysiology behind SCID. Which immune cells are affected?

T cells always affected. In some cases B and NK cells are affected too (Phenotypes: Always T-, then: B+ or B-, NK+ or NK-) Even patients with B cells normally have no/few antibodies due to lack of T-cell help

What is the global prevalence of tuberculosis?

TB is the ninth leading cause of death worldwide and the leading cause from a single infectious agent, ranking above HIV/AIDS.

Which have more genetic variation/diversity - TCRs or antibodies?

TCRs --- more possible J segments + D segments *This is surprising because .... the binding between TCR and antigen peptides is of relatively low affinity and is degenerate: that is, many TCRs recognize the same antigen peptide and many antigen peptides are recognized by the same TCR.

A 30-year old female is diagnosed with an E. coli urinary tract infection. Which is the single most critical mediator of acute inflammation in response to Gram- negative bacteria and other microbial organisms? A. Activated macrophages B. Chemokine CXCL8 C. Interferon gamma D. Interleukin-12 E. Tumour necrosis factor

TNF Activated macrophages are a sign that inflammation is occurring but are activated in the first place by other cytokines such as interferon gamma and tumour necrosis factor. CXCL8 or IL-8 is produced by macrophages and is involved in inflammation. Il-12 is also produced by macrophages and other immune cells in response to antigenic stimulation. Tumour necrosis factor is the key underlying cytokine that activates and controls other cytokines and cells in the inflammatory and so is the best answer.

What do the accessory proteins vip, vpr, vpu & nef in HIV do?

Take out the antiviral systems in host cells.

Factors affecting viral infectivity

Temperature Ionic environment and pH Lipid solvents, detergents, bleach Ionizing radiation

What allows apoptopic cells to be gotten rid of/phagocytosed?

The LOSS OF MEMBRANE SYMMETRY is recognised & so they are engulfed by phagocytes

What is the "avidity" or "functional affinity" of an antibody? How does avidity compare to intrinsic affinity?

The avidity is the binding strength when antibody attaches to >1 identical epitopes on the antigen. The binding is STRONGER when the antibody is bound to more identical epitopes on the antigen!

Given the thickness of the Mycobacterium cell wall (impermeable to Gram stain), how can you see this bacilli under a microscope?

The bacilli can be readily seen using a fluorochrome stain but this requires a microscope with ultraviolet light

T-cell activation and immunological synapse: What is the immunological synapse?

The contact area between the T cell and the antigen-presenting cell, which many TCR molecules move to. This increased conc. of TCR molecules leads to tyrosine phosphorylation of ITAMs on CD3 chains ----> which ultimately leads to the ACTIVATION OF TRANSCRIPTION FACTORS NF-AT is an example of one of the transcription factors that gets activated. Fun fact: doctors use cyclosporine & FK506 as "immunosuppressants" to inhibit activation of NF-AT to block transplant rejection!

Which of the following statements is TRUE of receptor editing? A Has been described for antibody but not TCR B Is associated with reexpression of RAG-1 in germinal centers C Can involve V genes which are 3′ of the initially selected V gene D Involves the S (switch) sequences E For B-cells only occurs in bone marrow

The correct answer is B. Following immunoglobulin gene rearrangement in the bone marrow, B-cells switch off expression of RAG-1 and RAG-2. However, mature B-cells in germinal centers can reexpress both RAG-1 and RAG-2, thereby permitting receptor editing. Receptor editing has been described for both antibody and TCR genes. V(D)J rearrangement will have deleted all of the V genes 3′ of the one that was selected to recombine with a D segment (for the heavy chain) or a J segment (for the light chain). S sequences are associated with each of the immunoglobulin heavy chain constant region genes (except C δ) and are involved in class switching. The term receptor editing is used to refer to the replacement of an already rearranged V gene with a new V gene. While the initial rearrangement of the immunoglobulin genes only occurs in the bone marrow, receptor editing can occur elsewhere in locations such as the germinal centers in lymph nodes.

Which of the following is a complement component that is strongly chemotactic for neutrophils? A C9 B C5a C C3 D C3b E C5b

The correct answer is B. C5a is a powerful chemotactic agent and also an anaphylatoxin. C9 is the terminal complement component which forms part of the membrane attack complex. C3 has no direct biological action but is split to give C3a and C3b when the complement system is activated. C3b opsonizes microorganisms for adherence to phagocytic cells. C5b initiates formation of the membrane attack complex.

What is a major target of anti-fungal drugs?

The ergosterol in fungal cell membranes *but toxic due to similarity to human cholesterol!

What is the incubation period?

The interval between infection and the onset of disease.

What is the main site of antigen entry into the body?

The mucosal surfaces constitute the main site of antigen entry into the body. More than 50% of all lymphoid tissue is MALT

Epitope

The precise part of the ANTIGEN that the antibody recognises

Why might pig organs (xenografts) not be an ideal solution the paucity of human organs for transplant?

The problem of natural human antibodies to sugars (gal-alpha) expressed on the surface of pig cells has been solved by genetic modification (pigs have been genetically modified to remove these sugars), but concerns remain regarding the feasibility of xenotransplantation.

What is the RSS?

The sequence (that comes before the V,D, and J gene segments) that the recombinase enzyme recognises! Knows to start recombination process here

What is intrinsic "affinity" in terms of antigens?

The strength with which an antigen-binding arm (the Fab region) binds an antigenic epitope

1. Describe the "attachment" phase of viral replication 2. What is the Influenza virus attachment protein, and what receptor does it bind to?

The virus attachment protein binds to a specific receptor on the cell membrane- glycoprotein. Influenza- virus attachment protein (Haemagglutinin- HA) on surface of virion binds to virus receptor (sialic acid) on cell

What do follicular dendritic cells in the germinal centre of the lymph node do?

These cells present antigen to B cells (not by MHC)

Describe the 4 phases of the primary antibody response 1.Lag 2. Log 3. Plateau 4. Decline

These cellular events take time and result in a lag period before significant levels of antibody are detectable. After a couple of days increasing amounts of antibody are made as more and more B-cells divide and give rise to plasma cells. This leads to the log phase as the antibody increases in the circulation. There then follows a plateau phase in which the antigen has bound to and removed the free antigen so that the lymphocytes are no longer stimulated. Thus the antigen is removed and the antibody response subsides leading to a decline of antibody in the serum (Fig 7). Most of the plasma cells die after carrying out their function but some are long lived.

Possible uses of vaccines:

They are designed to induce a specific immune response which can: - Prevent infection (Polio) - Control existing infection (Zoster) - Prevent disease development post-exposure (Rabies) - Prevent foetal infection after immunisation of mother (Rubella) - Prevent or control cancer (HPV/HBV)

What mechanism are used by MTB to evade the immune response?

They can survive inside macrophages! i. They get phagocytosed, but then resist degradation in the lysosome & then they ESCAPE from the ENDOSOME into the cytoplasm! And replicate there

What virulence characteristics of this organism contribute to avoiding the host defense mechanisms?

Thick waxy coat helps protect against acidity of stomach (surfactant bile salts are not powerful enough to wash away their waxy coats )

Pyelonephritis

This is a urinary tract infection (UTI) of the kidney. Cystitis (UTI of the bladder) is much more common and may affect up to 60% of women during the course of their lifetime. In certain instances, if left untreated, the infection can ascend from the bladder to infect the kidney and cause pyelonephritis. Pyelonephritis often presents with fever or chills, flank or lower back pain and often nausea or vomiting.

How is M. tuberculosis diagnosed in the laboratory?

To diagnose ACTIVE TB you need TISSUE e.g. sputum sample & they do a Ziehl-Nielsen Stain on it - and the mycolic acid will get stained ◦ Auramine (fluorescent) is a more sensitive stain for TB, but it requires a UV microscope a. Chest X ray : pulmonary TB b. Extrapulmonary TB? TB that occurs outside the lungs - can use: i. CT scan, MRI, Ultrasound of the affected part of the body, endoscopy, urine & blood tests, biopsy (small sample of tissue or fluid taken out) c. Mantoux test (TST) for LATENT TB: if small red bump forms at site of PPD tuberculin injection, you have latent TB. d. IGRA (Interferon Gamma Release Assay) **NOT USEFUL because can't distinguish between latent & active TB

What is a reason we require multiple signals (not just 1) to elicit a immune response?

To ensure we don't react to something that is not a pathogen. Need multiple signals for lymphocyte activation e.g. adaptive immunity w/ innate immunity, B cells w/ T cells etc

What are Koch's 5 postulates?

To prove an agent causes a disease, we must: 1. Find in lesions. 2. Isolate in pure culture. 3. Inoculate pure culture & cause disease. 4. Recover again from lesions of host.

Which receptor is the main cell surface receptor that binds LPS which forms a complex with our host cell surface proteins to activate signalling? How can LPS, present in gram-negative bacterial cell walls, lead to sepsis?

Toll-like receptor 4 Lipopolysaccharide (LPS), also commonly known as endotoxin, is a large molecule of a lipid and polysaccharide found in the outer membrane of gram-negative bacteria. LPS binds to receptors on many different immune cells promoting secretion of pro-inflammatory cytokines. In sepsis these pro-inflammatory signals are dysregulated and cause organ dysfunction.

How is toxoplasmosis spread? What are the 3 stages of toxoplamosis?

Toxoplasmosis is most commonly acquired from cat litter or uncooked meat BUT IT CAN ONLY BE transmitted transplacentally --> (must screen pregnant women, c.f. syphilis) 1.oocyst (resistant stage in external enviro) 2.tachyziote (proliferative form which invades macrophages - gives flu-like symptoms) 3.tissue cyst (resistant & latent in the brain but the cyst gets reactivated in the immunosuppressed -> cerebral abscess)

There is little or no antibody memory response in the absence of Th cells. T/F? As soon as a B cell meets antigen, the antigen selects the specific B cells to take part in the antibody response. What 4 things then happen to the selected B cells?

True 1. clonal proliferation (the selected B-cells undergo rapid clonal proliferation in order to expand up the number of antigen-specific cells to a sufficient level to deal with the infecting pathogen) 2. memory cells made (Some of the cells of the expanding clone become memory cells) 3. differentiation into plasma cells (which secrete large amounts of specific antibody). 4. class switching (Th cell interaction with the B cell also induces class switching - the B cell switches from producing one isotype to another.)

What is "true latency" ? + examples Incomplete latency? Chronic infection?

True latency: alternates between latent and productive infections. Infectious virus disappears after first infection but reappears (recurrence), sometimes many years later, e.g. herpes simplex virus (cold sores), varicella-zoster virus (chickenpox, shingles), in which neurones are the site of latency. Incomplete latency: latent in some cells but productive infection in others e.g. HIV Chronic infection: continuous productive infection e.g. hepatitis B & C

Viruses are the organisms with the most genetic diversity. T/F? Which 2 processes contribute to the massive diversity of viruses? *What is the difference between RNA-dependent RNA polymerases and DNA-dependent DNA polymerases in terms of diversity of RNA vs. DNA viruses?

True. Viruses have greater genetic diversity than any other group of organisms. 1. misincorporation of nucleotides & recombination (drift) 3. reassortment (shift) 1. POINT mutations. In every viral infection a small number of virus particles replicate to produce millions of progeny and errors in copying from the viral nucleic acid inevitably occur. Thus any virus population contains a mixture of point mutations. RNA-dependent RNA polymerases have no proof reading mechanisms so any errors become immortalised. Mutation rates are as high as one misincorporation/103 to 104 nucleotides polymerised. RNA virus populations are therefore a complex mixture of many non-identical but related viruses known as a QUASI-SPECIES NB 1. In uncontrolled HIV infection about 109 virions are produced/day and selective pressures on the HIV quasispecies predispose to the emergence of anti-viral drug resistant mutants and evasion of the immune response. NB 2. Point mutations are responsible for antigenic drift in influenza viruses. Antigenic drift has happened when virus particles with a slightly altered surface protein (antigenic) emerge as a result of point mutations and evasion of the immune response. [see Respiratory Virus Infections lecture in Circulation & Breathing module]. 2. Reassortment= = the exchange of entire RNA molecules between genetically related viruses with segmented genomes. Reassortment is responsible for antigenic shift in influenza viruses; a major change in one or more of the virus surface proteins and their antigenicity can lead to an influenza pandemic * DNA-dependent DNA polymerases have proof reading - exonuclease error correction. The misincorporation is about 10,000-fold lower than the RNA-dependent RNA polymerases..

Which parasite changes its VSG genes

Trypansoma burcei - antigenic polymorphism every week! >1600 VSG genes

What are the immunological events leading to a fatal episode of peanut allergy ?

Type I hypersensitivity. IgE-mediated mast cell degranulation i) Type I hypersensitivity ii) TH2 cells (or possibly THf (follicular) cells) drive IgE production ii) IgE binds Fcepsilon leading to mast cell degranulation iii) systemic mast cell degranulation leads to release of histamine, leukotrienes and systemic inflammation and vasodilation, and hence low blood pressure and death. Examples of type 1: • Upper respiratory tract; rhinitis, hayfever • Lower respiratory tract; atopic asthma. Asthma is the most common chronic disease of children in Western countries. Causes 2,000 deaths/year in UK • Skin : allergic eczema • Mouth, throat, gut : food (e.g. fish). • Systemic : peanuts • Infections : worms

Goodpasture's syndrome - what other condition is the same type of hypersensitivity?

Type II (Antibody to cell surface antigens) Patients with Goodpasture's syndrome have antibodies against both lung parenchyma & kidney basement membrane because these organs share common antigens. Haemolytic disease of the newborn

Systemic Lupus (SLE)

Type III autoantibodies against NUCLEAR components - immune complexes deposited over whole body, especially kidneys --> glomerulonephritis

What are the immunological events leading some farmers to become short of breath when working with hay ?

Type III hypersensitivity. i) Farmer's lung; type III hypersensitivity ii) breathing in moldy hay or grass leads to exposure to high levels of fungal and bacterial spores. iii) make IgG and IgM antibody to these antigens iv) On re-exposure, form immune complexes in lung, which activate complement and granulocyte infiltration. v) Cause immune reaction in and around alveolar spaces, which causes wheezing, shortness of breath, coughing etc. Most common moulds that cause Farmer's lung: Micropolyspora faeni" and "Thermoactinomyces vulgaris", and certain types of moulds called "Aspergillus" ARTHUS REACTION - antigen entering lungs which already have a high level of antibody triggers local tissue necrosis Normally phagocytic cells remove immune complex, and there is no tissue damage. But high levels of deposited immune complexes which do not get removed can cause damage to the whole system or to a local area (Arthus reaction). The immune complex activates complement via the classical pathway, with the subsequent recruitment of neutrophils (bottom left) by the chemotactic activity of C5a. The neutrophils then become attracted to the site (the lungs) and release lytic enzymes from their granules.

What are the immunological events leading to hair dye allergy ?

Type IV hypersensitivity. Delayed type (DTH); T cell mediated T-cell activation by antigen induces proliferation and cytokine release. Upon activation T cells of the Th1 type release cytokines such as IFN which stimulate macrophages to release inflammatory mediators such as TNF and reactive oxygen intermediates. e.g. contact dermatitis, Tuberculin skin reaction, poison ivy i) Allergy to hair dye components, eg paraphenyldiamine PPD ii) Type IV hypersensitivity ; allergic contact dermatitis iii) PPD binds proteins in skin, and after repeated exposure can drive T cell immunity iv Both CD4 and CD8 cells primed v) At subsequent exposure , CD4 and CD8 cells are recruited to the site of exposure (e.g. scalp) where they activate macrophages and mast cells, and cause local inflammation.

Fungal structure: - cell membrane - cell wall

Typical eukaryotic structure with nucleus containing chromosomes and cytoplasmic organelles Cell membrane contains ERGOSTEROL as the main sterol (c.f. cholesterol in the mammalian cell membrane) Cell wall consists of several types of polysaccharides (glucans, mannans), proteins and chitin Some fungi have melanin in the cell wall Some fungi have a polysaccharide capsule e.g. Cryptococcus neoformans

How is the skin different from the gut in terms of immunity?

Unlike the gut, the skin has very few RESIDENT immunological cells. Epidermis has very few resident cells - a few Langerhans (probing) and mainly keratinocytes making skin! But there are more immunological cells down in the dermis! And these are enough to alert the immune system that something is wrong. --- The dermis is highly vascularised, so it's v easy to recruit cells to the skin! Rather than rely on resident immunological cells, the skin primarily relies on the fact that it is a watertight barrier preventing pathogen entry.

When amplifying viral nucleic acid as a test, what 1st step do you need to make with viral RNA?

Use reverse transcriptase to convert it into cDNA

What is the treatment of M. tuberculosis infection?

Usual treatment: i. 2 antibiotics: Rifampicin & isoniazid for 6 months ii. Plus 2 extra antibiotics (pyrazinamide + ethambutol) for the first 2 months of this 6 month period **ethambutol can damage eyes Both pulmonary & extrapulmonary TB require a combo of antibiotics i. TB in areas like brain or sac surrounding heart (pericardium) may require CORTICOSTEROIDS also -- to reduce swelling Latent TB: usually treated unless old, because treatment can cause liver damage in old ppl. Latent TB will defo be treated in immunocompromised ppl as there is a higher chance their latent TB will become active Side effects of these antibiotics? 1. Isoniazid can cause nerve damage (peripheral neuropathy) - so take B6 alongside to reduce this risk 2. Rifampicin can make the contraceptive pill ineffective

Dermatophyte infections

V common: affect 1 in 3 ppl e.g. ringworm - utilize keratin as a nutrient source e.g. Trychophyton rubum + lots of Tinea moulds

What are mycoplasmas?

V. small, require sterols for growth, and lack a rigid cell wall = which means they can't be treated with penicillin! (p works by interfering w synthesis of cell wall) Mycoplasma pneumoniae causes bronchopneumonia

What is VSG? Which parasite is this found on?

Variant Surface Glycoprotein VSG shields all the other surface proteins from being recognised by the host & constantly changes its surface antigen so host cannot keep up Found on Trypanosoma brucei

Spirochaetes - why are they difficult to see under microscope & how would you detect them instead?

Very slender, tightly coiled spirals are hard to see so use lab tests looking for antigens instead! - syphillis is caused by spirochaetes

What are arboviruses?

Viruses which are transmitted via arthropods from animals to humans ( a type of zoonotic virus). --transmitted by means of an infected, blood-sucking, arthropod vector (ARthropod BOrne = arboviruses). Diseases caused by arboviruses include encephalitis, febrile diseases (sometimes with an associated rash), and hemorrhagic fevers.

Candida pathogenisis - hyphal switching

When pH goes above 7, hyphal switching is induced - so the yeast now becomes hyphae and is more invasive - it infects mucosal surfaces *common in women who have taken antibiotics which have reduced the presence of lactobacilli in the vagina which were previously keeping the environment acidic *always a fine balance between colonisation & infection, but the hyphal switching induces the infection

At what point does the B cell start creating SOLUBLE (secreted) version of antibody, rather than the transmembrane version of antibody (BCR)?

When the B-cell differentiates into a plasma cell --> starts secreting soluble antibodies!

What do interferons do?

When they bind to their receptor, they ACTIVATE a general ANTI-VIRAL state in a cell - so they INTERFERE with viral replication

Most abundant cells in bone marrow?

White blood cells

Wound dehiscence

Wound ruptures along surgical incision e.g. persistent with from an abdominal wound

T regs: a. express what protein on their surface & what transcription factor? b. name 3 of their mechanisms of action c. What is the name of the syndrome T-reg deficient humans have?

a. CD25 & Foxp3 b. - Secrete inhibitory cytokines (e.g. IL-10, TGFb) - Use the regulatory protein CTLA-4 to modify APC (block or remove costimulatory ligands) - Bind and consume IL-2 (a growth factor for T cells) c. IPEX: Immune dysregulation polyendocrinopathy enteropathy X-linked syndrome (mutations in Foxp3 gene) >80% IPEX patients have Type 1 diabetes (also IBD, allergy) --> early death

Distinguish between: a. Labile cells b. Stable (quiescent cells) c. Permanent + examples

a. Labile cells: constantly divide e.g. skin epithelial cells, gut epithelial cells, bone marrow cells b. Stable (quiescent cells) : cells that have a low level of replication under normal circumstances but can switch to rapid replication w the correct stimulus! e.g. epithelial cells of kidney, liver & other organs; fibroblasts; smooth muscle c. Permanent : incapable of mitosis or organised replication! e.g. neurones, skeletal & cardiac muscle

a. How do we monitor efficacy of HAART in patients? b. Why does HAART use at least 3 drugs in combination?

a. Measure viral load = amt of viral RNA in plasma b. To minimize the possibility of mutations 3 drugs: (e.g. NRTI, NNRTI &amp; protease inhibitor) also: at sub-maximal doses to minimise toxicity

How does the host respond against TB? (3)

a. PRRs recognize PAMPs --> triggers signaling cascade in alveolar macrophages -> resulting in macrophages phagocytosing the microbe and secreting pro-inflammatory cytokines e.g. TNF b. Neutrophils also play a defensive role, not only as first-line non-specific phagocytes, but also by secreting anti-bacterial proteins, c. NK cells, which are large granular circulating lymphocytes, are attracted to the sites of bacterial infections, where they specialize in recognizing and destroying infected host cells. During this process they secrete interferon gamma (IFNγ), which activates macrophages, inducing them to secrete the cytokines IL-12, IL-15 and IL-18, which activate CD8+T-cells, thus forming the link to the adaptive immune system [7,16]

a. Enfurvitide b. Maraviroc

a. Prevents FUSION of HIV envelope w/ cellular membrane - binds to GP41 on envelope glycoprotein to prevent conformational change needed for fusion b. Prevents HIV binding to the co-receptor (CCR5) on the surface of T cells (not just CD4 needed) to stop ATTACHMENT

In what way is the diagnosis of hepatitis B infection usually made and what would you understand by the term confirmation in this context?

a. Serology - detection of 3 types of HBV antigen: i. Surface antigen ii. Core antigen iii. "E" antigen (secreted by liver cells) b. All patients will have core antigen (HbcAg) and will make core antibody! c. IgM antibody tells u infection was recent = acute! d. Surface antigen will go undetected in "window" phase cos it is battling it out with the antibody --- so equal levels e. Surface antigen tells u yes/no if infected but not acute or chronic f. HBV DNA testing on the other hand is true marker of HBV replication and infection g. Anti-HBe antibody present in ppl who have had or who cleared the infection in the past

Pathogens - which immune cells are most important to protect you against each of these? a. viruses b. mycobacteria c. bacteria d. fungi

a. T cells & NK cells b. T cells & macrophages c. B cells & neutrophils d. T cells & neutrophils

Bone-marrow and haematopoietic stem cell transplantation has been successfully used for reconstitution of normal phagocytic function in ..(a).......... and of B and T cells in .......(b)...

a. chronic granulomatous disease (CGD) b. SCID

How is M. tuberculosis spread?

a. spread when a person with active TB disease in their lungs coughs or sneezes and someone else inhales the expelled droplets, which contain TB bacteria. (must spend hours in contact w someone tho) • Once at their preferred site of infection, pathogenic mycobacteria tend to allow themselves to be phagocytosed(swallowed) by Macrophages(white blood cells), within which they are able to resist destruction, and are able to multiply

Of the exotoxins that Gram Positive bacteria secrete, describe the function of a.) Neurotoxins b.) Enterotoxins c.) Cytotoxins

a.) Interfere with neurotransmitter activity e.g. Botulinum toxin can cause flaccid paralysis; Tetanus can cause spastic paralysis b.) Cause overactivity of secretory mechanisms c.) directly poison cells e.g. Shiga toxin destroys ribosomes

Transmission of HIV is increased a.) if the person at risk has genital herpes (HSV-2) b.) by male circumcision

a.) if the person at risk has genital herpes (HSV-2) - will have ulcers so epithelium is not intact- so easier for virus to cross the epithelial tissue and get into lymph nodes *male circumcision DECREASES risk of HIV - foreskin has high density of Langerhans & CD4 cells, so if that is not there, there's less opportunity for the HIV to infect

Which cancers are associated with the schistosome parasite?

bladder cancer

Comminuted fracture

bone is splintered usually from high impact

Di George syndrome

born without a thymus gland - so you don't have T cells

Which cancers are associated with liver flukes (parasite)?

cholangiocarcinoma

HBeAg antigen present after 6 months indicates....

chronic infection of HBV

Which bacteria might you find in the eye, and why are there few bacteria here in general?

eye = inhibitory environment due to lysozyme but Gram-positive rods (corynebacteria) may be found

VZV morphology

icosahedral capsid (has envelope) DS DNA (herpesvirus group - CHHEZ - CMV - Herpes Simplex 1 & 2 - EBV - VZV) Baltimore group 1

Main causes of cellular senescence?

include DNA damage, telomere erosion, reactive oxygen species (ROS) and activation of oncogenes.

X-linked lymphoproliferative disease

involves mutation in either the SH2D1A gene encoding an intracellular signalling molecule or the XIAP gene encoding an inhibitor of apoptosis. These patients are particularly susceptible to infection with Epstein-Barr virus.

Mortality rate

is expressed as the ratio of number of deaths from a disease in a given year to the total population at mid year.

Viruses are so small that they are BELOW the resolution of a .....

light microscope = "Submicroscopic" infectious agents

Streptococcus pyogenes under microscope

long chains of Gram positive cocci

Difference btwn lymph nodes & spleen in terms of which pathogens they trap?

lymph nodes trap antigens which come through lymphatics; spleen traps the antigens which come through the blood

Which 4 cells come from the lymphoid progenitor? & the myeloid progenitor?

lymphoid: B, T, NK and dendritic cells myeloid: neutrophils, macrophages, dendritic cells, monocytes (macrophaes in tissue)

Explain the phrase "All viral genomes lead to mRNA" ?

mRNA can be translated into proteins on host ribosomes, and viruses need to produce proteins innit

When B cells have differentiated into plasma cell precursors in the germinal centre, where do they go to develop into plasma cells?

medulla of the lymph nodes or red pulp of the spleen

Symptoms of meningitis

non-blanching rash, photophobia, fever, headache, limb/joint pain, confusion

Which respiratory infections can viruses like influenza, parainfluenza and respiratory syncytial virus (RSV) cause?

potentially fatal infections of the lower respiratory tract, such as bronchiolitis and pneumonia

Bacteria found in GI tract?

principally facultative and strict anaerobes e.g. upper GI tract - enterococci, coliforms (e.g. Escherichia.coli), lactobacilli, streptococci. Lower GI tract - principally anaerobes (Gram positive - Clostridium; Gram negative - Bacteroides spp), enterococci, streptococci, coliforms

Meningitis rash progression What would happen if you press a glass against the skin of the patient?

rash: 1. early petechial (1-2mm capillary bleeding) rash 2. classic purpuric (3-10mm bleed) rash - a later sign of serious disease As the infection progresses, there are increased purpuric lesions with areas of haemorrhage and necrosis The rash is non-blanching - doesn't disappear when you press a glass against it.

What is the most common type of primary immunodeficiency?

selective IgA deficiency

Which gene defects may cause SCID?

sex-linked: mutations in the X- linked common gamma chain used by several cytokine receptors autosomal mutations: - encoding adenosine deaminase (ADA), - the recombination-activating genes RAG-1 and RAG-2, - the JAK-3 signalling molecule, - the Artemis component of the DNA-dependent protein kinase complex.

What transports the 'Dimeric' secretory IgA across the mucosal barrier ?

the poly Ig receptor

MALT (collection of lymphoid cells): These may occur as diffuse collections of lymphocytes, plasma cells, dendritic cells and macrophages throughout the lung and the lamina propria of the intestinal wall, or as more clearly organized tissue with well-formed follicles? which are the more organized tissues with well-formed follicles?

tonsils and the small intestinal Peyer's patches

As well as a combined antigen/antibody test, what other markers might be looked at to diagnose HIV?

viral genome - DNA &/or RNA load & sequencing (only case where might look at DNA is with babies)

What are the smallest infectious agents?

viruses (prions are even tinier though!)

HIV Structure? Which receptor (along with CD4 receptor) is absolutely necessary for entry of the virus into the cell?

• Lipid Envelope • 2 single strands of positive sense RNA : only 9 genes which make 15 diff proeins ◦ CCR5 co-receptor - needed for entry

What virulence characteristics of this organism contribute to avoiding the host defence mechanisms?

• M protein is the most important virulence factor. • M protein binds factor H - factor H inhibits complement pathways by DEGRADING C3-CONVERTASE (prevents opsonization, inflammation and cell lysis by MAC) • Also, by binding fibrinogen, M protein blocks deposition of C3b. • GAS produces a protease C5a peptidase, which INACTIVATES C5a (a powerful chemoattractant for neutrophils and macrophages). • They have many adhesins on their surface e.g. lipoteichoic acid • They have a hyaluronic acid capsule - this is antiphagocytic.

How is Strep. pyogenes diagnosed?

• Microscopy - infected tissues/body fluids can be examined under a microscope with gram stain. • Culture - swabs from the posterior pharynx/tissue/pus/blood can be grown on blood agar (24-48 hours on blood agar at 37 degrees and 5% CO2) to show the colonies with large zones of beta haemolysis. • Antigen detection - rapid tests detecting the A carbohydrate antigen on throat swabs. A negative test must be confirmed by culture. • Serology can also be used (to test for antibodies)

T cell diversity is produced by the same methods as B cell diversity except....

• Multiple germ line genes • V-J and V-D-J recombination • Recombinatorial inaccuracies • N-nucleotide addition • Chain combinations • BUT NOTTTTT Somatic hypermutation

What is the treatment of Strep. pyogenes infection?

• Penicillin • If patients are allergic, erythromycin or a cephalosporin can be used. • In cases of necrotising fasciitis, clindamycin with surgery is used for treatment. Also • Antibiotic treatment of GAS pharyngitis prevents the development of rheumatic fever. • In patients who have had rheumatic fever, prophylactic antibiotics are needed for several years.

Difference between primary & secondary immunodeficiency? Which one is more common?

• Primary are genetic (rare) • Secondary are acquired - HIV, chemotherapy, radiation, leukaemia, malnutrition Secondary is more common.

Distinguish between viruses that cause short term infections & viruses that cause persistent infections

• Viruses that cause short term infections require efficient transmission & are usually spread by aerosol or the faecal-oral route. Virus is shed in high titre • Viruses that cause persistent infections do not require such highly efficient modes of transmission because they are shed continuously or intermittently for many years e.g. VZV, HIV

Symptoms of TB

◦ Cough for more than 3 weeks w/phlegm (may be bloody) ◦ Weight loss, night sweats, fever, fatigue, loss of appetite, neck swelling


Kaugnay na mga set ng pag-aaral

"Light Speed" and "The Milky Way"

View Set

Chapter 13: Supply Chain Process Integration

View Set

check your understanding 44, 45, 47, 51, 52

View Set

105E.1 Building Blocks of the Human Body

View Set

biceps brachii, brachialis, coracobrachialis, triceps brachii

View Set

Life Span Development by John Santrock 13th Edition Chapter 8

View Set

Lesson 12: LIFE IN A PURITAN COMMUNITYLIFE IN A PURITAN COMMUNITY

View Set

Anatomy and physiology directional terms

View Set

Chapter 6 concepts & Definitions

View Set

Chemical Formulas & Equations (Test 12/9)

View Set